Hand - Limbs - Congenital Flashcards

1
Q

A 5-month-old infant presents with nail-patella syndrome. Which of the following is the most likely associated genetic anomaly in this patient?

A) Bone morphogenetic protein (BMP)
B) Engrailed-1 (EN1)
C) Fibroblast growth factor receptor-2 (FGFR2)
D) LIM homeobox transcription factor 1 beta (LMX1B)
E) Sonic hedgehog (SHH)

A

The correct response is Option D.

The most likely anomaly is in LIM Homeobox transcription factor 1 beta (LMX1B).

This patient presents with nail-patella syndrome, which results in poorly developed nails and kneecaps. Nail-patella syndrome is an autosomal dominant condition associated with mutations in the LMX1B gene. This gene is involved in dorsal and ventral limb patterning during embryology.

Limb development is controlled by signaling pathways that are located within three different signaling centers. These govern the proximal-distal, anterior-posterior, and dorsal-ventral axes of limb development.

Wnt7a from the dorsal ectoderm induces the transcription factor Lmx-1b, which induces the development of dorsal structures. This transcription factor acts within the Wnt signaling pathway, which is responsible for dorsal-ventral limb development. In the ventral portion of the limb, the Engrailed-1 gene product blocks the Wnt pathway, determining the dorsal-ventral boundary.

Mutation in the FGFR2 gene can result in Apert syndrome, which is an autosomal dominant condition that presents with complex syndactyly of hands and feet.

Sonic hedgehog acts within the zone of polarizing activity, which is located in the posterior margin of the limb bud and governs growth in the anterior-posterior axis. The sonic hedgehog protein signals development of the limb into radial and ulnar aspects. This is mediated by the Gli family of transcription factors, most importantly Gli3. Alterations in this pathway can result in mirror duplication.

Bone morphogenetic protein (BMP) is actively involved in multiple areas of upper extremity development, but is not the key anomaly involved in this syndrome. Suppression of BMP is associated with syndactyly.

How well did you know this?
1
Not at all
2
3
4
5
Perfectly
2
Q

A 5-year-old boy presents with Blauth/Manske Grade IIIa hypoplasia of the thumb, with narrowing of the first web space, hypoplastic thenar muscles, an unstable metacarpophalangeal joint, and a stable carpometacarpal (CMC) joint. Examination shows weakness of grasp of his toys and utensils, and he has failed a 6-month course of therapy. Which of the following is the most appropriate surgical approach to improving the patient’s grasp?

A) Abductor digiti minimi transfer
B) Free great toe transfer
C) Fusion of the thumb interphalangeal joint
D) Pollicization of the index finger
E) Thumb CMC ligament reconstruction with tendon interposition arthroplasty

A

The correct response is Option A.

Blauth developed the classification for thumb hypoplasia, which consists of five grades. Manske expanded the classification system by expanding the definition of grade III into grade IIIa, which has a carpometacarpal (CMC) joint, and grade IIIb, which does not:

Blauth I thumbs have minimal shortening and narrowing, and they do not require surgical treatment.

Blauth II thumbs have narrowing of the first web space, hypoplastic thenar muscles, and an unstable metacarpophalangeal (MCP) joint. They are best treated with reconstruction.

Blauth/Manske IIIa thumbs have type II features, extrinsic tendon abnormalities, and metacarpal hypoplasia with a stable CMC joint. They do well with reconstruction.

Blauth/Manske IIIb thumbs have type IIIa features plus partial metacarpal aplasia and CMC joint instability. Because of the additional complexity of CMC joint reconstruction, thumb ablation and pollicization is generally recommended.

Blauth IV thumbs are often referred to as pouce flottant, or floating thumb. There is a neurovascular bundle coursing through the stalk of skin connecting the thumb to the hand, and these thumbs have few rudimentary bony or soft-tissue structures. These thumbs are best treated with ablation and pollicization.

Blauth V dysplasia demonstrates absence of the thumb and require pollicization.

Blauth II and Blauth/Manske IIIa reconstructive options include the abductor digiti minimi (ADM or Huber) transfer and the flexor digitorum superficialis (FDS) transfer. There is no clear indication of the superiority of one over the other. The use of the ADM diminishes the power of abduction of the small finger but provides some thenar bulk. It is a better pronator of the thumb. The FDS transfer removes a flexor from the ring finger, perhaps decreasing grip strength, and fails to provide any bulk to the thenar eminence. The FDS is superior in providing palmar abduction but pronates less effectively. When additional tissue is needed to stabilize the MCP joint, the FDS can also be used to reconstruct the MCP joint as part of the transfer.

Aside from pollicization, additional reconstructive options for Blauth IIIb-V dysplasia include second toe-to-hand transfer with metatarsalphalangeal (MTP) joint arthrodesis if parents and patients decide to keep a five-digit hand. Due to the level of aplasia, harvest of the MTP joint is necessary for anatomic reconstruction. The second and third toes are best used for this level of reconstruction due to the ability to harvest the MTP joint. The great toe cannot be used, as harvest of the MTP joint of the great toe will lead to gait and balance issues. Fusion of the thumb interphalangeal joint and thumb CMC ligament reconstruction and tendon interposition arthroplasty do not address any or all of the issues with the thumb dysfunction.

How well did you know this?
1
Not at all
2
3
4
5
Perfectly
3
Q

A 4-month-old male infant is brought to the office with the multiple extremity findings shown in the photographs. Which of the following additional clinical findings is most commonly seen with this condition?

A) Absence of the sternocostal head of the pectoralis major
B) Bilateral symmetric anomalies
C) Clubfoot
D) Craniosynostosis
E) Hematopoietic changes

A

The correct response is Option C.

This child has constriction ring syndrome (CRS). CRS is a rare occurrence, with the incidence reported from 1 in 1200 to 15,000 births. CRS frequently affects multiple extremities asymmetrically, with an average of three extremities involved, most often at the distal aspect of the extremity. Several characteristics are most often seen in the affected extremities including constriction rings, amputations, lymphedema distal to the ring, and acrosyndactyly (digits are fused distally, but proximally have fenestrations as fusion occurred after apoptosis in utero). Amputation or hypoplasia is usually seen distal to the ring, whereas the proximal aspect is intact and usually well-formed. CRS affecting the hands most frequently affects the index, middle, and ring fingers. Associated anomalies include syndactyly, oligodactyly, clubfoot, limb absences, body-wall defects (e.g., gastroschisis), and craniofacial anomalies, particularly facial clefts.

Symbrachydactyly, which is associated with Poland syndrome, is usually present in one limb. There is true hypoplasia of the affected extremity with absent or rudimentary digits with nubbins and frequent syndactyly. Symbrachydactyly usually impacts the central digits. Poland syndrome is defined by the absence of the sternocostal head of the pectoralis major.

Hematopoietic changes can be seen in radial hypoplasia. Fanconi anemia and thrombocytopenia (thrombocytopenia absent radius syndrome) are both associated with radial hypoplasia.

Children with Apert syndrome (acrocephalosyndactyly) have bilateral relatively symmetric complex syndactyly, symphalangism, and craniosynostosis.

How well did you know this?
1
Not at all
2
3
4
5
Perfectly
4
Q

The lower extremity difference known as mirror foot is most likely linked to which of the following derangements in embryologic development?

A) Duplication of the apical ectodermal ridge
B) Duplication of the zone of polarizing activity
C) Loss of the apical ectodermal ridge
D) Loss of the signal from ventral ectoderm
E) Loss of the zone of polarizing activity

A

The correct response is Option B.

The zone of polarizing activity (ZPA) is responsible for differentiation in the anteroposterior axis. Regulation of this entity involves the sonic hedgehog and Hox b-8 proteins. Loss of the ZPA results in absence of posterior elements in the developing limb. Duplication of the ZPA leads to mirror hand and mirror foot type anomalies.
The apical ectodermal ridge (AER) is the primary driver for proximal-distal limb growth. The AER is highly regulated by fibroblast growth factor, and loss of AER function leads to arrested limb development.
Ventral (bone morphogenetic proteins and Engrailed-1) and dorsal (Wnt7) signaling factors are antagonistic; disruption of these signals will affect dorsal-volar patterning but also proximal-distal growth of the limb.

How well did you know this?
1
Not at all
2
3
4
5
Perfectly
5
Q

During index finger pollicization for congenital absence of the thumb (Blauth type V), the first dorsal interosseous muscle of the index finger is used to create the function of which of the following muscles?

A) Abductor pollicis brevis
B) Abductor pollicis longus
C) Adductor pollicis
D) Extensor pollicis longus
E) Flexor pollicis longus

A

The correct response is Option A.

Thumb hypoplasia represents a spectrum of thumb deficiency that may range from mild generalized hypoplasia to complete absence of the thumb. The modified Blauth classification is useful not only for classifying thumb deficiency, but also for guiding management decisions. Index finger pollicization is the procedure of choice for Blauth thumb hypoplasia type IIIB, type IV, and type V.

In pollicization, the index finger is transposed on its neurovascular bundles. The index metacarpal is removed from its base to the epiphysis. The new carpometacarpal joint (CMC) of the thumb is constructed by preserving the metacarpophalangeal joint of the index finger, which is fixed into hyperextension (to avoid hyperextension deformity of new CMC joint). The index metacarpal is positioned in 45 degrees of abduction and 100 to 120 degrees pronation and fixated anterior to the index metacarpal base.

The index finger tendons and muscles are used to construct the functions of the new thumb:
-The extensor digitorum communis is sutured to the base of the new metacarpal to simulate the abductor pollicis longus.
-The extensor indicis proprius tendon is shortened and sutured to become the new extensor pollicis longus.
-Further stability is achieved by transfer of the previously mobilized index dorsal interosseous to the radial lateral band to restore abductor pollicis brevis function.
-The index palmar interosseous is inserted into the ulnar lateral band to construct the adductor pollicis.

How well did you know this?
1
Not at all
2
3
4
5
Perfectly
6
Q

A 4-month-old male infant is evaluated because of a deep constriction band of the right lower leg present since birth. There is marked edema of the leg and foot distal to the ring, which has increased substantially over the past several weeks. Which of the following is the most appropriate management for this patient?

A) Amputation of the lower extremity
B) Application of a compression garment
C) Band excision
D) Collagenase injection
E) Limb elevation and observation

A

The correct response is Option C.

Amniotic band syndrome is a rare congenital anomaly with a sporadic inheritance pattern and a reported incidence that ranges between 1:1200 to 1:15,000 live births. Constriction bands may range from shallow, harmless skin depressions to deep, annular bands that may present with distal edema, neurovascular compromise, or autoamputation. There is a propensity for constriction bands to affect the more distal aspect of the upper and lower limbs (including fingers and toes) in a bilateral, asymmetric fashion. Associated conditions may include: acrosyndactyly, talipes equinovarus (clubfoot), and cleft lip/cleft palate.

In this patient, constriction band excision is the most appropriate management given the increasing edema and its potential associated functional issues. Different strategies have been described for band excision and closure. Although extremity band excision was traditionally performed in two stages, one-stage resection has been performed safely by some surgeons.

Limb elevation and observation is not the best treatment as it will not address the underlying cause of the worsening edema.

Compression garments and collagenase injections do not have roles in the treatment of extremity constriction bands.

Amputation is not the best option and would be considered only in an unsalvageable extremity.

How well did you know this?
1
Not at all
2
3
4
5
Perfectly
7
Q

A 3-month-old female infant diagnosed with Apert syndrome is referred to the office for evaluation of both hands. Which of the following clinical findings is most likely to be present?

A) Camptodactyly
B) Complex syndactyly
C) Floating thumb
D) Macrodactyly
E) Ulnar clinodactyly of the thumb

A

The correct response is Option B.

Apert syndrome (acrocephalosyndactyly) is a syndrome associated with an FGFR2 gene mutation and characterized by craniosynostosis, mid-face hypoplasia, and complex syndactyly of the hands and feet. Characteristics of the Apert hand deformity include complex syndactyly of the index, middle, and ring fingers; simple syndactyly of the ring and small fingers; and a varying degree of first web space syndactyly. The Apert hand deformity has been classified into three subtypes on the basis of the increasing degree of thumb involvement and the appearance of the central digital mass. In the Apert hand deformity, the thumb may be short and radially deviated (radial clinodactyly).

The other responses are not associated with Apert syndrome. Floating thumb (“pouce flottant”) is a form of thumb hypoplasia (Blauth Type IV) in which there is a rudimentary digit attached to the hand only by skin and neurovascular bundle.
Camptodactyly is a painless flexion contracture of the proximal interphalangeal joint most commonly involving the small finger.
Macrodactyly is a rare congenital condition in which there is overgrowth of the underlying bone and soft tissue of the finger.

How well did you know this?
1
Not at all
2
3
4
5
Perfectly
8
Q

A 12-month-old boy comes to the office for evaluation of a congenital thumb deformity. A photograph is shown. Which of the following is the most appropriate management of the thumb?

A) First metacarpal distraction osteogenesis
B) First web space deepening
C) Flexor digitorum superficialis opponensplasty
D) Free great toe transfer
E) Index finger pollicization

A

The correct response is Option E.

This patient has a type IV (floating thumb or pouce flottant) thumb hypoplasia best managed with thumb ablation and index finger pollicization. Thumb hypoplasia can range from a minor decrease in size to complete absence. Thumb hypoplasia may also be associated with VACTERL (vertebral abnormalities, anal atresia, cardiac abnormalities, tracheoesophageal fistula or esophageal atresia, renal agenesis and dysplasia, and limb defects), TAR syndrome, Fanconi anemia, and Holt-Oram syndrome. Given the possible association with these conditions, a workup of all potentially affected organ systems should be performed. This includes a renal ultrasound, an echocardiogram, spine x-ray studies, a complete blood count, and a chromosomal challenge test.

Thumb hypoplasia may occur in an isolated fashion or in combination with radial hypoplasia, as seen in this case. The most important and difficult distinction is between types IIIa and IIIb hypoplastic thumbs. A type IIIa thumb has a stable thumb carpometacarpal (CMC) joint, whereas a type IIIb thumb does not. If the child uses the thumb for integrated pinch, the CMC joint is deemed stable. If the thumb is bypassed in favor of scissor pinch between the index and middle fingers, the CMC joint is either unstable or absent; in this case, a pollicization is recommended.

A free great toe transfer is not indicated in children with thumb hypoplasia because the existing thumb CMC is not stable and pollicization would provide a better functional outcome. Flexor digitorum superficialis or abductor digiti minimi opponensplasty are both options for thumb hypoplasia reconstruction in the presence of a stable CMC joint. A first webspace deepening is typically combined with opponensplasty and metacarpophalangeal joint stabilization in children with reconstructible hypoplastic thumbs, but not for children undergoing pollicization. The patient in the image does not have a first metacarpal, and therefore cannot undergo distraction osteogenesis.

How well did you know this?
1
Not at all
2
3
4
5
Perfectly
9
Q

A 9-month-old boy presents for evaluation of unilateral preaxial polydactyly. X-ray studies show triphalangism of the accessory thumb. Which of the following is the most likely Wassel classification in this patient?

A) Type II
B) Type III
C) Type IV
D) Type V
E) Type VII

A

The correct response is Option E.

Preaxial polydactyly describes patients with thumb duplication. In 1969, Wassel (as the fellow of Adrian Flatt) described a categorization system for radial polydactyly corresponding to the level of skeletal duplication. There have been many subsequent modifications of this classification system, but this remains the most commonly used. Type IV (duplicated proximal and distal phalanges) is the most common duplication, followed by Type II (duplicated distal phalanx). A Type VII duplication involves a triphalangeal thumb and is the only deformity in which there is triphalangism.

How well did you know this?
1
Not at all
2
3
4
5
Perfectly
10
Q

A 2-year-old male infant presents with a congenital deformity of the ring finger. A photograph and x-ray study are shown. The patient’s parents report that the condition limits his ability to make a full fist but is otherwise painless. Which of the following is the most likely diagnosis in this patient?

A) Amniotic band syndrome
B) Camptodactyly
C) Congenital trigger finger
D) Macrodactyly
E) Syndactyly

A

The correct response is Option A.

Amniotic band syndrome (ABS) has an incidence of 1/1200 to 1/15,000 births. Some congenital anomalies have been associated with ABS including cleft palate, imperforate anus, equinovarus, and body wall defects.

The etiology of ABS has two theories. The intrinsic defect theory endorses genetic abnormalities which lead to mesenchymal hypoplasia and scarring. The extrinsic theory endorses amniotic tissue which entangles fetal parts leading to constriction.

Constriction varies leading to a spectrum of clinical manifestations from skin dimpling to digital amputation. Neurovascular injury can manifest as peripheral nerve palsy, lymphedema and arterial insufficiency. Syndactyly is common, and acrosyndactyly is pathognomonic of ABS. The Patterson classification system has four subtypes. The first is a simple constriction ring. The second has a constriction ring that affects the digit distal to the ring, with or without lymphedema. The third consists of constriction rings with acrosyndactyly. The fourth is characterized by amputation at any level.

Camptodactyly is defined as a painless and progressive non-traumatic contracture of the proximal interphalangeal (PIP) joint. It affects around 1% of the population and the great majority of the cases are extremely mild and asymptomatic. The cause of the contracture is controversial. There have been descriptions of malformations of the superficial flexor of the fingers, lumbrical muscles, and the transverse and oblique retinacular ligaments. There may also be alterations to the configurations of the PIP joint.

Congenital trigger finger differs from congenital trigger thumb. Congenital trigger finger is rare and presents more commonly in the ulnar digits with associated malformations of the superficial and deep flexors. It presents with sporadic locking. Release of the A1 pulley alone is not adequate, with tenoplasty of the chiasm and partial opening of the A2 pulley generally necessary.

Syndactyly is a variable fusion between two adjacent fingers, and is one of the most common congenital
deformities, occurring in 1:2000 live births.
Classification includes:
Simple: fusion only through the skin
Complex: bone connection.
Complete: the entire commissure is involved, including the nail bed
Incomplete: the nail bed is not involved.
Complicated: involvement of vascular tissues, tendons or nerves.

It can occur separately or as a manifestation of a syndrome, such as Streeter, Apert or Poland, in which the severity of the syndactyly is more significant.

Macrodactyly is a congenital overgrowth disorder and represents 0.9% of upper extremity congenital anomalies. Digital enlargement involves all tissue types and maintains patterns of growth and anatomic relationships within the affected portion of the hand.

The term “macrodactyly” is reserved for nonsyndromic, congenital enlargement of a digit or digits that occurs in isolation without associated limb hemihypertrophy or vascular anomaly.

How well did you know this?
1
Not at all
2
3
4
5
Perfectly
11
Q

Which of the following failures in embryologic development is most likely to have caused the deformity pictured in the photographs shown?

A) Differentiation of the zone of polarizing activity
B) Formation of the apical ectodermal ridge
C) Inhibition by en-1
D) Longitudinal formation
E) Programmed cell death

A

The correct response is Option E.

The hand plate initially forms with webbed digits. In order to have separate fingers, the interdigital tissue must undergo programmed cell death/apoptosis. Bone morphogenetic protein (BMP) plays an essential role in this process. A complex interplay creates failure of longitudinal formation (i.e., radial club hand). Removal of the apical ectodermal ridge (AER) results in limb truncation. The zone of polarizing activity (ZPA) is present in the posterior aspect of the developing upper limb and helps direct the anteroposterior axis (radial-ulnar axis). Sonic hedgehog (SHH) is the critical signaling factor. ZPA transplantation or excess SHH results in mirror hand deformity. The dorsoventral axis is another pathway critical for appropriate limb development. The WNT7A signaling molecule is expressed in the developing upper limb dorsal ectoderm, activating the LIM homeodomain, resulting in the expression of LMX1B transcription factor from the dorsal mesenchyme, whereas the ventral ectoderm induces the expression of en-1. These signaling factors are necessary for the formation of dorsal versus palmar structures of the hand.

How well did you know this?
1
Not at all
2
3
4
5
Perfectly
12
Q

A 6-month-old male infant presents with bilateral radial longitudinal deficiency and thumb hypoplasia. X-ray study is shown. Which of the following studies is most likely to rule out Fanconi anemia in this patient?

A) Bone marrow biopsy
B) Chromosomal breakage testing
C) Complete blood count with differential
D) Peripheral blood smear
E) Renal ultrasonography

A

The correct response is Option B.

All children with radial longitudinal deficiency (RLD) should undergo a thorough physical examination because of the frequency of associated syndromes. Common conditions associated with RLD include thrombocytopenia-absent radius (TAR) syndrome, Holt-Oram syndrome, VACTERL syndrome and Fanconi anemia.

Thrombocytopenia-absent radius (TAR) syndrome is an autosomal recessive disorder characterized by an absent radius with a relatively normal thumb. Holt-Oram syndrome is an autosomal-dominant disorder in which RLD is accompanied by either an atrial or ventricular cardiac septal defect. Fanconi anemia is an autosomal-recessive pancytopenia, once invariably fatal, that is now routinely treated with bone marrow transplantation. One should consider further screening with spinal x-ray studies, cardiac echocardiography, renal ultrasonography, and complete blood count.

Because children with Fanconi anemia often do not manifest aplastic anemia on routine blood testing until after age 3 years, it is also recommended that patients with RLD undergo a chromosomal breakage test for earlier detection of this potentially fatal disease. Routine genetic counseling is also recommended. A peripheral blood smear and bone marrow biopsy would not be helpful in making this diagnosis.

How well did you know this?
1
Not at all
2
3
4
5
Perfectly
13
Q

An 18-month-old male infant with an isolated Blauth Type IV thumb (floating) is scheduled to undergo pollicization of the index finger. A photograph is shown. The index finger metacarpal head becomes which of the following structures in the pollicized thumb?

A) Articular surface of the carpometacarpal (CMC) joint
B) Distal articular surface of the proximal interphalangeal (PIP) joint
C) Proximal articular surface of the PIP joint
D) Scaphoid
E) Trapezoid

A

The correct response is Option A.

Thumb pollicization is a challenging yet functionally rewarding procedure. It is indicated for Blauth Type IIIB or higher thumbs. If done correctly, this technique can greatly enhance the function of the hand and provide a strong, stable thumb analog for grip and pinch. The procedure involves many stages and is technically demanding. Although beyond the scope of this description, the procedure requires shortening of the index finger metacarpal and repositioning of the metacarpal head in hyperextension against the preserved metacarpal base. Thus, the index metacarpal phalangeal joint becomes the new thumb’s carpometacarpal joint, the index proximal interphalangeal joint becomes the new thumb’s metacarpophalangeal joint, and the index distal interphalangeal joint becomes the new thumb’s interphalangeal joint.

How well did you know this?
1
Not at all
2
3
4
5
Perfectly
14
Q

A newborn is diagnosed with a hypoplastic thumb and an anorectal malformation. These findings are most likely associated with which of the following conditions?

A) Fanconi anemia
B) Fetal alcohol syndrome
C) Holt-Oram syndrome
D) Thrombocytopenia-absent radius syndrome
E) VACTERL association

A

The correct response is Option E.

Radial longitudinal deficiency may present as an isolated condition but is often associated with an underlying syndrome. The frequency of association with a syndrome has been reported to range from 33 to 44%. Hypoplastic thumb can be present in isolation or in combination with any radial deficiency. Hypoplastic thumb is the second most frequently encountered thumb anomaly. Commonly associated syndromes with radial longitudinal deficiencies include Holt-Oram, Fanconi anemia, thrombocytopenia-absent radius, and VACTERL association (a sporadic collection of anomalies consisting of the vertebral deformity, anal atresia, cardiac anomalies, tracheoesophageal fistula, renal agenesis, and limb deformities).

Holt-Oram is a syndrome passed in an autosomal dominant fashion and presents with both radial longitudinal deficiency and a cardiac anomaly, most commonly a ventricular septal defect.

Both thrombocytopenia-absent radius syndrome and Fanconi anemia are transmitted in an autosomal recessive pattern. Thrombocytopenia absent radius syndrome consists of thrombocytopenia that manifests itself during infancy and can be fatal; however, it usually resolves spontaneously with age. In contrast, Fanconi anemia presents after 3 years of life, commonly around 8 to 9 years, with aplastic anemia. Historically this condition was fatal, with no method of altering the course of the anemia. Recently, however, bone marrow transplants have been performed successfully to treat the anemia and prolong the life expectancy of these patients. Lastly, the final condition seen commonly with radial longitudinal deficiency is VACTERL association. Anorectal malformations (ARM) often co-occur with upper limb anomalies, mainly of pre-axial origin. ARM patients with a major upper limb anomaly—with or without other congenital anomalies—have a twofold greater chance of a genetic disorder than have non-isolated ARM patients without upper limb anomalies. Not all upper limb anomalies in ARM patients are part of the VACTERL association.

Fetal alcohol syndrome causes brain damage and growth problems, however it is not associated with thumb hypoplasia.

How well did you know this?
1
Not at all
2
3
4
5
Perfectly
15
Q

A 12-month-old male infant is brought to the office for evaluation of a small right thumb. Physical examination shows instability of the metacarpophalangeal joint and hypoplasia of the first web space. The carpometacarpal joint is stable. The patient has difficulty moving the thumb. Which of the following is the next best step in management?

A) Free toe transfer
B) Metacarpal lengthening
C) Opponensplasty
D) Pollicization
E) Continued observation

A

The correct response is Option C.

This patient presents with a Blauth Grade 2 or 3A thumb hypoplasia. Surgical management is warranted and includes treatment of the absent or hypoplastic thenar musculature with opponensplasty; release of the first web space; and management of MCP instability.

Pollicization is reserved for Blauth Grade 3B or more severe deficiencies. The distinction between a Blauth 3A and 3b hypoplastic thumb is determined by the stability of the CMC joint.

Blauth classification of Thumb Hypoplasia:

How well did you know this?
1
Not at all
2
3
4
5
Perfectly
16
Q

A 2-month-old infant is brought to the office for evaluation of bilateral complex syndactyly of the hands and feet. Which of the following is the most likely gene involved?

A) Engrailed-1 (EN1)
B) Fibroblast growth factor receptor-2 (FGFR2)
C) LIM Homeobox transcription factor-1 beta (LMX1B)
D) Sonic hedgehog (SHH)
E) Wingless type (Wnt)

A

The correct response is Option B.

This patient presents with complex syndactyly of hands and feet consistent with Apert syndrome. This is an autosomal dominant condition that can be caused by mutation in the **fibroblast growth factor receptor-2 gene (FGFR2).

Limb development is controlled by signaling pathways that are located within three different signaling centers. These govern the proximal-distal, anterior-posterior, and dorsal-ventral axes of limb development.

The limb develops in a proximal-distal direction, from shoulder to wrist, which is controlled by the apical ectodermal ridge (AER), a thickened layer of ectoderm over the limb bud. Fibroblast growth factors secreted within the AER signal the underlying mesoderm to differentiate. Disruption of the AER results in truncation of the limb.

Growth in the anterior-posterior (radioulnar) axis is determined by the zone of polarizing activity (ZPA). This is located in the posterior margin of the limb bud. The sonic hedgehog protein (SHH) acts to signal development of the limb into radial and ulnar aspects. Alterations in this pathway can result in mirror duplication.

Dorsal-ventral limb development is signaled by the Wnt signaling pathway, which produces a transcription factor, LMX1B, that induces the development of dorsal structures. Deficiency in LMX1B has been associated with nail-patella syndrome. In the ventral portion of the limb, the Engrailed-1 gene product blocks the Wnt pathway, leading to ventralization.

A variety of other transcription factors encoded by Hox and T-Box genes also govern limb and organ development, and alterations in these can give rise to various developmental differences.

How well did you know this?
1
Not at all
2
3
4
5
Perfectly
17
Q

Which of the following is the greatest predictive risk factor for birth brachial plexus injury?

A) Cesarean delivery
B) Forceps delivery
C) Macrosomia
D) Multiple gestation
E) Shoulder dystocia

A

The correct response is Option E.

In multivariate analysis, shoulder dystocia has overwhelmingly been found to be the risk factor most predictive of (most associated with) birth brachial plexus injury. The epidemiological study by Foad showed a 100 times greater risk; another study by DeFrancesco showed an odds ratio of 113.2. While conferring a smaller risk, macrosomia and forceps delivery are far weaker predictive factors, with increases in risk of 14-fold and 9-fold, respectively. Multiple gestation and cesarean delivery are actually protective factors against birth brachial plexus injury.

How well did you know this?
1
Not at all
2
3
4
5
Perfectly
18
Q

Which of the following techniques is most commonly used for reconstruction of a Blauth IIIB thumb hypoplasia in the United States?

A) Great toe to thumb transfer
B) Index pollicization
C) Metacarpophalangeal (MCP) joint stabilization with opponensplasty
D) Second toe to thumb transfer
E) Toe wrap around transfer

A

The correct response is Option B.

Blauth classification of Thumb Hypoplasia:

The most severe forms of thumb hypoplasia (types IIIB, IV, and V) require amputation of the existing thumb and reconstruction, most commonly by index pollicization.

How well did you know this?
1
Not at all
2
3
4
5
Perfectly
19
Q

Congenital constriction band syndrome is responsible for which of the following percentages of all congenital upper limb defects?

A) 6%
B) 12%
C) 18%
D) 24%
E) 30%

A

The correct response is Option B.

Constriction band syndrome and amniotic band sequence are the terms applied to a wide range of congenital anomalies, most typically limb and digital amputations and constriction rings which occur in association with fibrous bands. These classic syndromic birth defects represent disruptions and do not occur along the known lines of embryologic development. It has been proposed that the birth defects are caused by the action of the fibrous amniotic bands in association with a rupture of the amnion at early stage of pregnancy, which was later referred to as the “extrinsic theory.” In this theory, low amniotic fluid level plays a major role in the development of constriction rings. This has become the more widely accepted theory. However, many cases are associated with birth defects not readily explained by the mechanism of fibrous strings entangling body parts and causing disruption of fetal structures. A subset of cases manifests with cleft lip and palate (CLP), congenital heart defect, and renal anomalies. There are also case reports of children presenting with polydactyly, supernumerary nipples, and skin tags suggesting a genetic origin.

Based on a study on 419 upper limb defects and 171 lower limb defects occurring among 753,342 births in Finland during 1993 to 2005, constriction band syndrome comprises approximately 12% of all congenital upper limb defects and 14% of lower limb defects. Other skeletal and non-skeletal anomalies were present in 30% of the affected children, suggesting a possible genetic etiology.

How well did you know this?
1
Not at all
2
3
4
5
Perfectly
20
Q

A newborn male is brought to the tertiary multidisciplinary referral center for evaluation of anorectal malformation, tracheoesophageal fistula and absent right thumb. Which of the following associated VACTERL diagnoses is the best predictor of inpatient mortality?

A) Aniridia with brain stem hypoplasia
B) Anomalies of spine or vertebrae
C) Cardiac disease
D) Renal or urinary anomaly
E) Tracheal stenosis with stridor

A

The correct response is Option C.

Anomalies of the spine or vertebrae (V), anorectal malformations (A), congenital cardiac anomalies (C), esophageal atresia/tracheoesophageal fistula (TE), renal and urinary abnormalities (R), and limb lesions (L) frequently co-occur and are recognized as VACTERL anomalies. VACTERL association is typically diagnosed in the presence of at least three characteristic features in the absence of evidence for an overlapping condition, and is estimated to occur in approximately 1 in every 10,000 to 40,000 live births. The presence of either anorectal malformation or esophageal atresia alone generally triggers a workup for associated VACTERL diagnoses because of their significant impact on morbidity and mortality. For example, in a large cohort of children undergoing surgical repair of anorectal malformations, Lautz et al. found associated VACTERL diagnoses including congenital heart disease in 40.4%, renal or internal urinary disease in 34.7%, spinal or vertebral anomalies in 31.4%, esophageal atresia/tracheoesophageal fistula in 7%, and limb defects in 5.6%. The most common limb defects in VACTERL association include poorly developed or missing thumbs, or underdeveloped forearms and hands, polydactyly, syndactyly, and reduction deformities of the lower limb.

Independent predictors of mortality in any patient with VACTERL association include congenital heart disease (greatest for those who require cardiac surgery than those with a diagnosis but no operation), birthweight < 2 kg, and black race. Of note, the association between cardiac disease and higher mortality has been reproduced in several studies.

Aniridia, brain stem hypoplasia, and tracheal stenosis with stridor are not primary characteristics of VACTERL association.

How well did you know this?
1
Not at all
2
3
4
5
Perfectly
21
Q

A 1-year-old boy is brought to the office by his parents for evaluation and correction of the findings shown in the photograph. The patient is otherwise healthy. Which of the following techniques is most appropriate to reconstruct the apex of the web space in this patient?

A) Bilobed flap from the dorsal hand
B) Cross-finger flap
C) Full-thickness skin grafting
D) Proximally based dorsal rectangular flap
E) Split-thickness skin grafting

A

The correct response is Option D.

Full-thickness skin grafts are usually used to close the resultant defects of the fingers after release of congenital syndactyly. Several local dorsal flaps have been described to reconstruct the web space with differing shapes to avoid the use of skin grafts, including rectangular, a V-Y island, a dorsal flap interdigitated with a small palmar V and a dorsal V-shape. A bilobed flap has been described for treating web creep in adults, but would not routinely be used.

How well did you know this?
1
Not at all
2
3
4
5
Perfectly
22
Q

A 14-month-old girl is evaluated for surgical management of the condition shown in the photograph. Which of the following is the most appropriate classification of this congenital difference?

A) Duplication
B) Failure of differentiation
C) Failure of formation
D) Overgrowth
E) Undergrowth

A

The correct response is Option C.

The image shown is a patient with a typical cleft hand, or ectrodactyly. This is classified as a Swanson type I (failure of formation) congenital difference of the upper extremity. This deformity is caused by a longitudinal growth deficiency of one or more central digits. This condition may have a genetic underpinning and is most commonly inherited in autosomal dominant fashion with approximately 70% penetrance.

Ectrodactyly presents with significant heterogeneity. Many patients with cleft hand do not require any type of surgical management. The most important determinants of surgical management are establishment of an appropriate thumb to index webspace for gripping and grasping and release of syndactyly. Surgical management includes elevation of skin flaps, transposition of the index finger to the ulnar side of the cleft, and creation of a web space between the thumb and index finger.

Cleft hand is classified with the Thumb-Index Classification of Central Deficiency proposed by Manske. This generally guides the treatment approach in conjunction with management of syndactyly. A myriad of techniques have been employed based upon the severity of the webspace contraction and the presence of syndactyly adjacent to the cleft.

How well did you know this?
1
Not at all
2
3
4
5
Perfectly
23
Q

A 2-year-old boy is treated for congenital constriction band of the left upper extremity with distal edema. The child undergoes excision of 50% of the circumferential constriction band with direct closure. This intervention is most likely to have which of the following effects on the edema of the distal limb?

A) Gradual improvement during childhood development
B) Gradual worsening during childhood development
C) Significant improvement within a few weeks of surgery
D) Significant worsening during childhood development
E) No effect

A

The correct response is Option C.

Congenital constriction band syndrome is a sporadic condition with an incidence varying from 1:1200 to 1:15,000 live births. There is a significant predilection for the upper extremities and distal limbs. The two main objectives for the treatment of congenital constriction band syndrome are improvement of function and improvement of cosmetic appearance. Different surgical techniques, such as Z-plasty, have been described and used for decades; however, direct closure after the excision of the constricting band is a simple yet effective treatment.

With complete circumferential constriction bands, it is recommended that a two-stage correction approach be used. At the first operation, one-half of the circumference is excised; the other half can be excised after 3 to 6 months. This will avoid any problems to the distal circulation in the limb, which may already be compromised. Edema, when present, will significantly improve within a few weeks of the first surgery.

How well did you know this?
1
Not at all
2
3
4
5
Perfectly
24
Q

A 6-month-old boy is brought to the office with a Blauth type IV (pouce flottant) thumb deficiency. Reconstruction with pollicization is planned. Which of the following is the most appropriate new function of the first palmar interosseous muscle?

A) Abductor pollicis brevis
B) Abductor pollicis longus
C) Adductor pollicis
D) Extensor pollicis longus
E) Flexor pollicis brevis

A

The correct response is Option C.

The most appropriate new function of the first palmar interosseous muscle is the adductor pollicis.

Thumb deficiencies can be categorized according to the Blauth classification, ranging from hypoplasia (type I) to complete absence (type V). In deficiencies of type IIIB and greater, pollicization of the index finger is the recommended treatment.

In pollicization, the index finger is transferred on its neurovascular bundle and a new thumb carpometacarpal (CMC) joint is created using the metacarpophalangeal joint of the index finger. The tendons and muscles of the index finger are used to recreate functions of the new thumb. The extensor digitorum communis is used for the abductor pollicis longus, the dorsal interosseus becomes the abductor pollicis brevis, and the palmar/volar interosseous becomes the adductor pollicis. The extensor indicis proprius is used for the extensor pollicis longus. The flexor tendon is not typically disturbed.

How well did you know this?
1
Not at all
2
3
4
5
Perfectly
25
Q

A 4-month-old infant is brought to the office for evaluation of a unilateral, seven-fingered hand with a central index finger flanked on each side by long, ring, and little fingers, with no thumb present. Abnormal expression of which of the following controlling substances is most likely responsible for this developmental anomaly?

A) Bone morphogenetic protein
B) Fibroblast growth factor
C) Sonic hedgehog protein
D) Wingless protein

A

The correct response is Option C.

The infant’s described anomaly is that of a mirror hand, which is an abnormality of the anteroposterior (radioulnar) axis of limb development. Sonic hedgehog protein (SHH) is expressed in the zone of polarizing activity (ZPA)on the posterior (ulnar) surface of the developing limb bud and is the main controlling substance for this axis of development. Direct expression of SHH results in development of the ulna, the ulnar two columns of the carpus, the little finger, the ring finger, and the ulnar half of the long finger. Diffusion of SHH, or long range signaling, results in development of the radial half of the long finger and the index finger. Absence of SHH results in the development of the radius, the radial column of the carpus, and the thumb. Ectopic expression of SHH, such as with duplication of the ZPA at the anterior (radial) margin of the limb bud, results in duplication of the ulnar-sided structures found in a mirror hand.

Fibroblast growth factors (FGF) are the main controlling substances in the apical ectodermal ridge (AER) and are responsible for the proximodistal axis of limb bud development. Bone morphogenetic proteins (BMP) are expressed in the AER and are also thought to be involved in digital separation, the lack of which leads to syndactyly. Wingless proteins (WNT) are also expressed in the AER, but it is the WNT signaling pathway from the dorsal surface of the limb bud that controls the dorsoventral axis of limb development. Dorsal ectodermal WNT also maintains SHH in the ZPA such that lack of WNT leads to lack of ulnar ray development. This is contrary to the ectopic expression of SHH leading to duplication of the ulnar rays in mirror hand. Fibroblast growth factors, bone morphogenetic proteins, and wingless proteins are not principal controlling substances for the anteroposterior (radioulnar) axis of development of the limb.

How well did you know this?
1
Not at all
2
3
4
5
Perfectly
26
Q

A neonate is born with unilateral edema of the arm. During the ensuing day, there is worsening edema and bullae formation. Which of the following is the most appropriate next step in management of this patient?

A) Administration of intravenous antibiotics and elevation of the arm
B) Compartment release
C) Continued observation and dressing change
D) Debridement and skin grafting
E) Treatment in a hyperbaric chamber

A

The correct response is Option B.

Compartment release should be performed early and urgently. The other treatments above may play a role in secondary treatment. Volkmann’s ischemia is not rare and awareness of the diagnosis is important. The sentinel lesion of neonatal compartment syndrome is bullous formation on the dorsum of the hand or arm. Etiology is often in utero and can be amniotic bands, umbilical cord loops, forceps extraction, oligohydramnios, preeclampsia, and excessive maternal weight. Diastolic pressures in neonates are usually 40 mmHg or less. Small increases in compartment pressure are therefore not tolerated well. The six P’s of compartment syndrome, which include pain, paresthesias, pulselessness, pallor, paralysis, and pressure, will not be seen in the neonate. The three A’s, which precede the six P’s in children, are agitation, anxiety, and increasing analgesic requirements; and will not be seen in the neonate.

How well did you know this?
1
Not at all
2
3
4
5
Perfectly
27
Q

A 3-year-old boy is scheduled to undergo the first stage of separation of syndactyly of the right hand. Photographs are shown. Which of the following anatomical structures is most likely to limit how far the digits can be separated proximally?

A) Artery
B) Extensor tendon
C) Flexor tendon
D) Nerve
E) Vein

A

The correct response is Option A.

Digital veins, due to their larger number and relative redundancy, do not normally limit which tissues can be separated to what level. Division of one branch of the digital artery is not normally a problem when only two digits are involved in syndactyly; the digital artery on the unaffected side is normally sufficient to perfuse the digit. For a central digit when three or more digits are involved, dissection must either stop at the bifurcation of the artery, or revascularization, such as with a vein graft, must be done.

The digital nerve often exists as a single unit at least partially down the length of the joined fingers, but it can be separated into two proper digital units with intrafascicular dissection to a level proximal to the neo-web space.

Flexor and extensor tendons are not involved in syndactyly.

How well did you know this?
1
Not at all
2
3
4
5
Perfectly
28
Q

During embryonic limb development, limb bud outgrowth is controlled by which of the following families of signaling proteins?

A) Bone morphogenetic proteins
B) Fibroblast growth factor
C) Retinoic acid
D) Sonic hedgehog (SHH)
E) Wingless-type proteins

A

The correct response is Option B.

The most appropriate answer is fibroblast growth factor. Limb bud development begins at the fourth week of gestation via an outgrowth of lateral plate somatic mesoderm covered by ectoderm. The lower limb bud forms a few days after the upper limb and is formed opposite of the L3-5 vertebrae. The cells at the tip of the limb bud ectoderm thicken to form the apical ectodermal ridge (AER). Fibroblast growth factor 10 (FGF10) from the mesenchyme induces the formation of the AER, which produces FGF8. This signaling induces mitosis of the underlying cells leading to limb elongation.

A group of mesenchymal cells on the posterior of the limb bud form the zone of polarizing activity (ZPA). Cells from the ZPA secrete sonic hedgehog protein, which acts as a short range signaling molecule. The ZPA is responsible for the anteroposterior axis formation of the limb bud.

Retinoic acid, a derivative of vitamin A, plays a role in the proximal-distal development of the limb but is not the primary signaling molecule. The precise mechanism and action of retinoic acid is debated. A retinoic acid gradient is necessary for normal limb development. Retinoic acid may act as an inhibitor of FGF8, which then allows forelimb budding to occur. Retinoic acid is not thought to be involved in hindlimb formation.

During limb development, the digits are initially connected by tissue that regresses via apoptosis. Apoptosis of interdigital tissue is dependent on BMP signaling under the influence of sonic hedgehog from the ZPA. Proteins from the wingless type family, especially Wnt7a, are responsible for the dorsal-ventral patterning of the overlying ectoderm. The presence of Wnt7a is necessary to dorsalize the limb

How well did you know this?
1
Not at all
2
3
4
5
Perfectly
29
Q

A 3-month-old male infant is evaluated for polydactyly and mirror hand. Which of the following is the most likely genetic anomaly in this patient?

A) Engrailed-1 (EN1)
B) Fibroblast growth factor-2 (FGF2)
C) LIM homeobox transcription factor 1-beta (LMX1B)
D) Sonic hedgehog (SHH)
E) Wingless type (WNT)

A

**The correct response is Option D.

This patient presents with polydactyly and mirror hand. The most likely genetic anomaly is in sonic hedgehog (SHH).

Limb development is controlled by signaling pathways that are located within 3 different signaling centers. These control the proximal-distal, anterior-posterior, and dorsal-ventral axes of limb development.

The limb develops in a proximal-distal direction, from shoulder to wrist, which is controlled by the apical ectodermal ridge (AER), a thickened layer of ectoderm over the limb bud. Fibroblast growth factors secreted within the AER signal the underlying mesoderm to differentiate. Disruption of the AER results in truncation of the limb.

Growth in the anterior-posterior (radioulnar) axis is determined by the zone of polarizing activity (ZPA). This is located in the posterior margin of the limb bud. The sonic hedgehog protein (SHH) acts to signal development of the limb into radial and ulnar aspects. Alterations in this pathway can result in mirror duplication.

Dorsal-ventral limb development is signaled by the Wnt signaling pathway, which produces a transcription factor, Lmx-1 that induces the development of dorsal structures. In the ventral portion of the limb, the Engrailed-1 gene product blocks the Wnt pathway, leading to ventralization.

A variety of other transcription factors encoded by Hox and T-Box genes also govern limb and organ development, and alterations in these can give rise to various developmental differences.

How well did you know this?
1
Not at all
2
3
4
5
Perfectly
30
Q

A 7-month-old male infant with complete syndactyly is evaluated for surgical release. Which of the following is the most appropriate technique for reconstruction of the web space?

A) Coverage with a distally based zigzag flap
B) Coverage with a dorsal rectangular flap
C) Coverage with a volar triangular flap
D) Full-thickness skin grafting
E) Split-thickness skin grafting

A

The correct response is Option B.

The most appropriate technique for reconstruction of the web space in patients with complete syndactyly is coverage with a dorsal rectangular flap.

In patients with compete syndactyly, separation does not typically result in sufficient skin to resurface both digits entirely. Reconstruction of the web space is a priority and is typically achieved with a proximally-based dorsal rectangular flap. Interdigitating skin flaps are designed, and skin grafts are placed to replace skin deficiency. Full-thickness skin grafts are preferred to split-thickness skin grafts because of the decreased likelihood of contracture; however, these grafts are not typically placed in the web space.

How well did you know this?
1
Not at all
2
3
4
5
Perfectly
31
Q

A 3-month-old boy is brought for evaluation because of bilateral simple syndactylies between the long and ring fingers. During which gestational age range did this developmental anomaly most likely occur?

A) Days 0-15
B) Days 20-35
C) Days 40-55
D) Days 65-80
E) Days 85-100

A

The correct response is Option C.

Syndactyly is the most common congenital limb malformation. Digital separation is the result of distal to proximal apoptosis, likely mediated by bone morphogenic proteins (BMP) signaling TGF-beta. Overexpression of BMP inhibitors and persistent fibroblast growth factor (FGF) function have been found to lead to syndactyly.

The exact dates of digital separation vary by publication, but occur approximately between days 44 and 54. Limb buds appear by day 26, and a hand paddle by day 33. By day 56, the phalangeal tips are ossified.

How well did you know this?
1
Not at all
2
3
4
5
Perfectly
32
Q

A 3-month-old male infant is brought to the clinic by his parents for evaluation of craniofacial and hand anomalies. Genetic testing confirms an FGFR2 abnormality consistent with Apert syndrome. Which of the following hand anomalies is most likely in this infant?

A) Camptodactyly
B) Clinodactyly
C) Ectrodactyly
D) Macrodactyly
E) Syndactyly

A

The correct response is Option E.

The hand anomaly most associated with Apert syndrome is syndactyly.

Apert syndrome, or acrocephalosyndactyly, is a syndrome resulting from a defect in chromosome 10q, the site of the fibroblast growth factor-2 receptor. This syndrome consists of coronal craniosynostosis, mid face hypoplasia with retrusion, and syndactylies of the hands and feet. These may be type I (syndactyly of digits 2–4 with a free thumb; “spade hand”), type II (syndactyly of digits 2–4 with a simple syndactyly of the thumb; “mitten hand”), or type III (complex syndactyly of all digits with complex thumb syndactyly; “rosebud hand”).

The other choices are not typically associated with Apert syndrome. Camptodactyly is a fixed-flexion deformity of the proximal interphalangeal (PIP) joint(s), most commonly the little finger. Macrodactyly involves the localized overgrowth of bone and soft tissue. Clinodactyly is congenital deviation of a digit in the radioulnar plane. Ectrodactyly refers to a split-hand/split-foot malformation.

How well did you know this?
1
Not at all
2
3
4
5
Perfectly
33
Q

A 52-year-old man is evaluated because of a congenital deficiency of the upper extremity. A photograph is shown. This patient’s developmental abnormality most likely occurred during which of the following periods of gestation?

A) 0–4 weeks
B) 5–8 weeks
C) 9–13 weeks
D) 14–18 weeks
E) 19–23 weeks

A

The correct response is Option B.

The most likely period of time during gestation for this abnormality is 31 to 45 days.

This patient presents with transverse deficiency of the upper limb (phocomelia). This can result from abnormal development of the limb with an interruption in longitudinal growth.

The limb develops with formation of a limb bud at 26 days after gestation, which rapidly enlarges with growth of the embryo until approximately 47 days. At around 52 to 53 days after gestation, the fingers are separate. Embryogenesis is complete by 8 weeks, and all limb structures are present.

Limb development is controlled by signaling pathways that are located within three different signaling centers. These control the proximal-distal, anterior-posterior, and dorsal-ventral axes of limb development.

The limb develops in a proximal-distal direction, from shoulder to wrist, which is controlled by the apical ectodermal ridge (AER), a thickened layer of ectoderm over the limb bud. Fibroblast growth factors secreted within the AER signal the underlying mesoderm to differentiate. Disruption of the AER results in truncation of the limb.

Growth in the anterior-posterior (radioulnar) axis is determined by the zone of polarizing activity (ZPA). This is located in the posterior margin of the limb bud. The sonic hedgehog protein acts to signal development of the limb into radial and ulnar aspects. Alterations in this pathway can result in mirror duplication.

Dorsal-ventral limb development is signaled by the Wnt signaling pathway, which produces a transcription factor, LMX1 that induces the development of dorsal structures. In the ventral portion of the limb, the EN1 gene product blocks the Wnt pathway, leading to ventralization.

A variety of other transcription factors encoded by Hox and T-Box genes also govern limb and organ development, and alterations in these can give rise to various developmental differences.

Most limb abnormalities occur during the period of 4 to 8 weeks. In this case, the transverse deficiency likely occurred earlier in the process of limb outgrowth, while abnormalities of the digit would have taken place towards the end of this period.

How well did you know this?
1
Not at all
2
3
4
5
Perfectly
34
Q

A 1-month-old male newborn is brought to the office by his parents for evaluation of complete simple syndactyly between the second and third toes of the left foot. The parents are concerned about the newborn’s development and ability to achieve normal ambulation. Which of the following is the most appropriate intervention at this time?

A) Surgical correction is optional but not necessary because there is no functional deficit
B) Surgical correction with a dorsal local flap and ligament reconstruction
C) Surgical correction with a dorsal local flap only
D) Surgical correction with a dorsal local flap, skin grafting, and ligament reconstruction
E) Surgical correction with skin grafting only

A

The correct response is Option A.

Syndactyly of the toes most commonly affects the second and third digits of the lower extremities. Various levels of webbing are possible, from partial to complete. In general, syndactyly is a condition that is primarily cosmetic and rarely requires treatment. If surgery is desired, it should be postponed until the child is old enough to take part in the decision-making process. Surgery is often performed to avoid emotional and psychological problems.

Surgical correction most commonly consists of a dorsal rectangular flap and simultaneous full-thickness or split-thickness skin grafts. However, some surgeons are performing syndactyly repair of both the upper and lower extremities with dorsal local flaps and allowing the skin defects to epithelialize in order to avoid donor-site morbidity of a skin graft. If there is only soft-tissue involvement, ligament reconstruction is not needed.

How well did you know this?
1
Not at all
2
3
4
5
Perfectly
35
Q

A full-term male newborn is evaluated because of Blauth Type IV (pouce flottant) thumb hypoplasia. Physical examination shows no other abnormalities. A pollicization procedure is planned. At which of the following ages is it most appropriate to perform this procedure?

A) 1 Month
B) 1 Year
C) 5 Year
D) 10 Year
E) 15 Year

A

The correct response is Option B.

Pollicization for thumb hypoplasia Types IIIB and IV, and for aplasia, has been successful when performed when the patient is between 3 months and 3 years of age.

Most surgeons will prefer to do pollicization procedures when the patient is between 6 and 12 months of age because there is a more natural integration and adaptation process in using the pollicized digit as a thumb while the child becomes more involved with manual activities. Nevertheless, the outcome also depends on the quality of the anatomical structures of the pollicized digit.

Technically, the procedure can be done at any time; however, at less than 3 months of age, the child’s cardiopulmonary maturation may be incomplete. Also, the child becomes aware of the thumb after the age of 3 months, so performing the procedure earlier than this age only raises the degree of technical difficulty because of the minute size of structures and potential circulatory issues, without any further advantage.

The older the child is at the time of the procedure, the more difficult it becomes to change the patterns of pinch and prehension that have already begun in the first 3 years of life. Fine coordination is achieved at 3 years of age. Reorganization of cortical representation of the index finger as a thumb is a slower and incomplete process in older children or adults. Exposure of the new configuration of the hand to the process of neuromuscular growth of the child is a key consideration.

How well did you know this?
1
Not at all
2
3
4
5
Perfectly
36
Q

A female newborn is evaluated in the neonatal intensive care unit because of severe congenital constriction band syndrome of the left lower extremity. Examination shows cyanosis and severe swelling of the distal affected extremity. The constriction is located at the distal thigh and is very deep, extending down to the anterior distal femur. A Doppler popliteal pulse is audible. Plantar stimulation shows no sensory pain withdrawal reflex. Which of the following is the most appropriate immediate treatment?

A) Application of medicinal leeches to the foot
B) Constriction excision and Z-plasties
C) Leg elevation and edema wrapping
D) Limb amputation
E) Microsurgical repair of popliteal artery and vein and of the sciatic nerve

A

The correct response is Option B.

The patient described has type IIIB constriction (amniotic) band syndrome. Constriction banding affects 1:1,200 live births. Severity of the banding is classified as follows:

Type I (mild) – Shallow indentation of skin and soft tissue without distal lymphedema;

Type II (moderate) – Distal lymphedema, acrosyndactyly, and even discontinuous neurovascular or musculotendinous structures, but without vascular compromise;

Type III (severe) – Progressive lymphaticovenous or arterial compromise;

Type IV – Intrauterine amputation.

Weinzweig introduced the concept of a dynamic, or type IIIB, subtype in which there is evolving vascular compromise and in which “limb rescue” can be performed with emergency surgery despite a severe band. The dynamic progressive swelling and cyanosis and certain ischemic necrosis can be reversed by surgery.

However, there are consequences, and these are neurologic deficits and long-term leg-length discrepancies that may even require leg-lengthening procedures or appropriately timed opposite extremity epiphysiodesis. Early release of such a severe constriction band does not result in long-term neurologic improvement. Early excision of the involved nerve segment may be required, accompanied by nerve grafting.

How well did you know this?
1
Not at all
2
3
4
5
Perfectly
37
Q

A 12-month-old boy is brought to the office for evaluation of complete absence of the right thumb. Examination shows no other extremity abnormalities. X-ray study confirms absent thumb metacarpal. Which of the following is the most appropriate method of reconstruction?

A) Great toe-to-thumb transfer
B) Huber transfer
C) Index finger pollicization
D) Osteoplastic procedure
E) Prosthesis

A

The correct response is Option C.

The most appropriate reconstruction of Type V deformity is pollicization of the index finger. It is the only satisfactory method of a basal joint reconstruction. Pollicization refers to the neurovascular pedicle movement of a finger, often with its metacarpal, for thumb reconstruction.

Great toe-to-thumb microsurgical reconstruction is appropriate when most of the first metacarpal is present. When the carpometacarpal (CMC) joint is absent or unstable, a great toe-to-thumb reconstruction will not reconstruct an adequate basilar joint.

A Huber transfer is used to create functional opposition when absent. Type IIIA deformities can be reconstructed with the Huber transfer, which involves transfer of the hypothenar muscle to recreate the thenar musculature.

Osteoplastic thumb reconstruction is rarely performed today because it results in a stiff, broad, floppy thumb with limited sensation. It involves the combination of a bone graft and flap to lengthen the thumb. At least three stages are required: bone graft from iliac crest covered in a tubed distant flap; flap pedicle division; and transfer of neurovascular sensory flap from the long finger to the thumb’s pinch contact surface.

A prosthesis would provide a stable post, but it would also lack sensation. The prosthesis will be more of an encumbrance and is not used in congenital thumb reconstruction.

How well did you know this?
1
Not at all
2
3
4
5
Perfectly
38
Q

A 36-year-old pregnant woman undergoes routine fetal ultrasonography. Examination of the embryo shows the formation of limb buds without digital separation. Based on the timeline of the development of the upper extremity, it is most likely that the fetus has reached which of the following points in gestation?

A) 2 Weeks
B) 5 Weeks
C) 8 Weeks
D) 11 Weeks
E) 14 Weeks

A

The correct response is Option B.

Embryonic development has been studied closely. The stages of human upper extremity development have also been studied closely. The limb bud is present without digital separation at 5 weeks. At 2 weeks of gestation, the limb buds have not yet formed. At 8 weeks, digital separation is nearly complete

How well did you know this?
1
Not at all
2
3
4
5
Perfectly
39
Q

An 8-year-old girl is brought to the office because her right breast has not developed like the left breast. Physical examination shows hypoplasia of the right breast, absence of the sternal head of the pectoralis major, and a congenital abnormality of the right hand. Which of the following categories of congenital abnormalities best describes this patient?s hand condition?

A) Constriction ring syndrome
B) Duplication
C) Failure of differentiation
D) Failure of formation
E) Overgrowth

A

The correct response is Option C.

The category of congenital abnormality that is most likely to be associated with the abnormality described is failure of differentiation.

In the patient with absence of the sternal head of the pectoralis major muscle, the diagnosis is Poland sequence. Poland sequence is associated with symbrachydactyly of the ipsilateral hand. A system of classification has been adopted by the American Society for Surgery of the Hand (ASSH) and the International Federation of Societies for Surgery of the Hand (IFSSH) in an attempt to provide a unified system to catalog congenital hand deformities. In this system, congenital hand differences have been categorized into seven types:

Type I – Failure of formation
The type I congenital deformities include scenarios in which there is a lack of formation. This includes longitudinal arrest, which can be found at any level from the shoulder to the phalanx, or transverse arrest with varying degrees of preaxial, central, or postaxial deficits, such as radial or ulnar clubhand.

Type II – Failure of differentiation
Failure of differentiation involves a variety of conditions with inadequate separation of parts, such as syndactyly, carpal coalitions and synostoses, camptodactyly, and clinodactyly. Symbrachydactyly, as seen in Poland syndrome, falls into this category.

Type III – Duplication
Duplication may apply to the whole limb or occur as mirror hand. It includes radial or ulnar polydactyly and triphalangeal thumb.

Type IV – Overgrowth
Macrodactyly and hemihypertrophy are types of overgrowth disorders.

Type V – Undergrowth
Undergrowth of the whole limb or parts of the hand may occur. Common manifestations include brachysyndactyly or brachydactyly.

Type VI – Constriction ring syndrome
Constriction ring syndrome may involve amputation at any level.

Type VII – Generalized skeletal deformities
These deformities include generalized syndromes, such as Apert syndrome, various craniosynostoses, and Holt-Oram/thrombocytopenia-absent radius (TAR) syndrome.

How well did you know this?
1
Not at all
2
3
4
5
Perfectly
40
Q

A 6-year-old boy is brought to the office by his mother because she is concerned about a deformity of the little finger of his right hand. She says the deformity has been present for approximately 3 years but has become progressively pronounced during the past year. On examination, the patient can flex the proximal interphalangeal joint but cannot extend it beyond the position shown in the x-ray. The little finger of his left hand is similarly affected, but the condition is less severe. Which of the following is the most likely diagnosis?

A) Camptodactyly
B) Delta phalanx
C) Kirner deformity
D) Symphalangism
E) Trigger finger

A

The correct response is Option A.

Camptodactyly is a congenital abnormality accounting for 5% of congenital hand anomalies. There is a flexion deformity that is often noted within the first year of life, but a second, noncongenital group occurs around age 10 years. It generally appears at the proximal interphalangeal (PIP) joint. It may occur as part of a syndrome but is often an individual defect. It is classified by the International Federation as Type II (failure of differentiation). It may be accompanied by subtle radiographic changes seen on the lateral view of the PIP joint with flattening of the circular surface of the proximal phalanx and an indentation in the neck of the proximal phalanx; dorsal positioning of the head of the proximal phalanx into the articulation of the base of the middle phalanx is also present.

A delta phalanx (or bracketed epiphysis) will result in clinodactyly. Clinodactyly is an excessive radial or ulnar angulation of the digit. Involvement of the little finger is common and often inherited as a dominant trait. It may be associated with a delta middle phalanx. If the deformity is severe enough, it may require surgical correction.

Kirner deformity is a progressive palmar and radial curvature of the distal phalanx of the little finger that presents in preadolescence. It is caused by an idiopathic, nontraumatic disruption of the epiphysis of the distal phalanx. It may be caused by cold exposure and frostbite and would affect multiple digits.

Symphalangism describes a congenital clinical stiffness of the PIP joints of the fingers. It is also included in the failure of differentiation classification category. Flexion creases are absent, and the fingers may be longitudinally foreshortened. Multiple digits are affected most commonly.

Trigger digits may occur in children but most commonly affect the thumb with a flexion deformity of the interphalangeal joint. They may occur with a flexion deformity of the PIP joints of the fingers, but the fingers are much less commonly affected than the thumb.

41
Q

A 2-year-old boy with VATER syndrome is brought to the office for evaluation because his parents are concerned that the thumb of the right hand appears small and weak. Physical examination shows weakness of intrinsic function of the right thumb. The flexor pollicis longus flexion crease is absent and the thenar eminence is flat. The left hand, including the thumb, appears normal. The patient uses a pincer-type motion between the right index and long fingers to pick up small objects. The ulnar collateral ligament at the metacarpophalangeal joint of the right thumb is lax. There is no gross dislocation at the carpometacarpal joint of the right thumb, but 6 mm of passive dorsoradial mobility is noted. An x-ray study and photograph are shown. Which of the following is the most appropriate management?

A) Amputation of the thumb and pollicization of the index finger
B) Flexor digitorum sublimis opponensplasty and ulnar collateral ligament stabilization
C) Huber transfer and ulnar collateral ligament stabilization
D) Tendon graft reconstruction of the flexor pollicis longus and ligament stabilization
E) Observation and occupational therapy until the child is age 4 to 5 years, followed by reassessment of function

A

The correct response is Option A.

The child described has a classic Blauth Type IIIB hypoplastic thumb. Although the carpometacarpal (CMC) joint does not dislocate, the subluxation described is consistent with the instability noted in Blauth Type IIIB thumbs. The photograph shows a thumb with a broad base, which is distracting to the examinee who is not familiar with this disorder. However, the thumb described has significant hypoplasia and pollex abductus. The x-ray study shows the classic finding of a narrowed and underdeveloped metacarpal base, the hallmark of CMC instability.

X-ray study is essentially diagnostic of the Blauth Type IIIB thumb. The most accepted treatment for this type of thumb is ablation/amputation of the existing thumb and replacement with the index finger in the thumb position (pollicization).

In the scenario described, the patient has no significant cortical representation or intentional control of the right thumb. The child picks up objects between the index and long finger, ignoring the thumb. No matter what maneuvers are undertaken to power and stabilize the existing thumb with tendon transfers, the child will still have little to no natural predilection to actually use the thumb. Although opponensplasty and ulnar collateral ligament stabilization are common solutions for Types II and IIIA thumbs, they would not be appropriate. These treatments are often combined with first web space deepening.

Treatment is recommended earlier than age 4 or 5 years to avoid resistance to the thumb ablation and pollicization, which can occur with time on the part of the parents and the child. The younger child will accommodate the pollicized digit better. Treatment of the hypoplastic thumb should be decisive and performed at the appropriate age, if possible.

Flexor pollicis longus reconstruction with a tendon graft will not improve thumb function in the scenario described. This type of surgery is not a standard therapy for hypoplastic thumbs in general. Ulnar collateral ligament stabilization is not recommended because the thumb requires pollicization.

42
Q

A 3-year-old girl is brought to the office because the second and third toes of both of her feet are webbed. The patient’s mother desires correction of the deformity. Physical examination shows a simple complete syndactyly. Which of the following is the most appropriate surgical treatment after release of syndactyly?

A) Local flaps
B) Local flaps and full-thickness skin grafts
C) Local flaps and split-thickness skin grafts from the lateral thigh
D) Primary closure

A

The correct response is Option B.

Syndactyly of the toes is a common congenital deformity of the lower extremity. Syndactyly of the toes is less of a functional problem than that of the hand, but it can represent a significant emotional and psychological toll on the patient and his or her family. Patients in cultures that are accustomed to open-toe footwear and sandals may have difficulty wearing shoes. Surgery is indicated for patients who desire correction of this deformity. Incomplete syndactyly of the toes can be repaired with local flaps without the use of grafts, but complete syndactyly requires the use of full-thickness skin grafts. Thinning of the fat from around the neurovascular bundles is essential to assure closure with local flaps. Skin grafts are seldom required for incomplete syndactyly. If they are required, full-thickness grafts should be obtained from the lateral groin. Full-thickness skin grafts harvested from the medial groin may produce undesirable hair growth as the child matures.

43
Q

A 5-day-old male newborn is evaluated in the neonatal intensive care unit because of hypoplasia of the thumb. His parents are distressed and would like him treated immediately. Physical examination shows Buck-Gramcko Grade V thumb hypoplasia. At which of the following ages is it most appropriate to perform reconstruction?

A ) 1 Month

B ) 1 Year

C ) 4 Years

D ) 10 Years

E ) 14 Years

A

The correct response is Option B.

A Buck-Gramcko Grade V or Type V thumb is the most severe form of thumb hypoplasia, as it constitutes an absence of the thumb. Therefore, the patient described will need a pollicization procedure. Though the literature is unclear as to the exact timing of such a procedure, it is recommended by Kozin that it be performed prior to the patient developing an undesired side-to-side pinch pattern between fingers, which is a compensatory mechanism that will develop after the first year. Furthermore, arguments have been made by Kleinman and Strickland to await the proper development of a newborn patient’s cardiopulmonary system, development of bimanual grasp, involution of embryonic endosteal circulation, and development of the digital vessels to a more substantial thickness. Each of these considerations will help to decrease complications in carrying out such a difficult procedure in a small hand. Therefore, the most appropriate timing would be at 1 year of age.

44
Q

A 1-year-old child is evaluated because of a radial polydactyly deformity with a duplicated distal phalanx of the thumb (Wassel Type II). Excision of the radial-sided thumb and reconstruction of the ulnar-sided thumb are planned. Which of the following is most likely to reestablish stability of the thumb?

A ) Early mobilization of the thumb 3 days after surgery

B ) Excision of the central bone and soft tissue (Bilhaut-Cloquet procedure)

C ) Pollicization of the index finger

D ) Reinsertion of the flexor pollicis brevis and abductor pollicis brevis muscles

E ) Use of a ligamentous and periosteal flap for radial collateral ligament reconstruction

A

The correct response is Option E.

Careful collateral ligament reconstruction in thumb duplication Types II and IV is critical in achieving stability during reconstruction. This is best done through the technique described by Manske, using a ligamentous and periosteal flap for reconstruction of the radial collateral ligament. In this technique, the radial thumb is excised, with care given to detaching the radial collateral ligament with a periosteal sleeve of tissue. The periosteal sleeve provides additional length to achieve a good repair of the radial collateral ligament.

Early mobilization of the thumb 3 days after surgery would likely be premature and may lead to destabilization of the reconstruction, rather than assisting with final stability. A thumb spica splint is typically used to immobilize such reconstructions for 4 to 6 weeks to aid healing.

Excision of central bone and soft tissue, commonly known as the Bilhaut-Cloquet procedure, can be used in Type II or Type IV duplications but would not be appropriate in the scenario described, in which the surgeon chooses to remove the radial-sided thumb and reconstruct the ulnar-sided thumb.

Pollicization of the index finger would not be appropriate at this level of duplication (Wassel Type II) because a very effective thumb can be made without resorting to the sacrifice of the index finger.

Reinsertion of the flexor pollicis brevis and abductor pollicis brevis muscles is not necessary in a Type II duplication, as the duplication is too distal (distal phalanx) to require any disinsertion or reinsert

45
Q

Which of the following congenital anomalies results from failure of differentiation? A ) Complex syndactyly B ) Congenital preaxial polydactyly C ) Long finger macrodactyly D ) Type 4 (pouce flottant variant) thumb hypoplasia E ) Unilateral below-the-elbow congenital amputation

A

The correct response is Option A.

The embryologic classification of congenital anomalies consists of seven groups of anomalies based on mechanism.

The first group of anomalies is categorized by congenital amputations resulting from transverse or longitudinal failure of formation. The proximal third of the forearm is most commonly affected, and in utero vascular insult is thought to cause these anomalies.

All polydactylies are categorized in the third group of anomalies: duplications. These also include mirror hand, radial (preaxial) polydactyly, and ulnar (postaxial) polydactyly.

Both simple and complex syndactyly fall into the second group of anomalies: failure of differentiation. Additional anomalies within this group include radioulnar synostosis, symphalangia, clinodactyly, and camptodactyly. This group can be thought of as having all of the requisite parts present but with incomplete separation of those parts.

Pouce flottant represents a Type 4 thumb hypoplasia and, like all variants of thumb hypoplasia, falls into the fifth group: undergrowth. The currently accepted classification of thumb hypoplasia was modified by Buck-Gramcko.

Macrodactyly is an overgrowth classification and not a disorder of differentiation.

46
Q

A 2-year-old girl has an unstable carpometacarpal (CMC) joint at the base of the thumb of the left hand. A photograph and x-ray study of the hand are shown. According to Blauth and Manske, which of the following is the most appropriate classification of this hypoplastic thumb?

A ) II

B ) IIIA

C ) IIIB

D ) IV

E ) V

A

The correct response is Option C.

The Blauth classification of the hypoplastic thumb, as modified by Manske, is a true working classification that can be used for planning treatment (see table). Type IIIB has an unstable CMC joint and is the €œcutoff point € for performing index finger pollicization and ablation of the hypoplastic thumb, versus attempting reconstruction of the original thumb.

47
Q

A 9-month-old infant is evaluated because of syndactyly. For which of the following is surgical intervention most likely to be contraindicated?

A ) Complex syndactyly related to constriction band syndrome

B ) Complex syndactyly related to Poland syndrome

C ) First web syndactyly related to Apert syndrome

D ) Simple syndactyly of the ring and little fingers

E ) Web syndactyly related to Type I super digit

A

The correct response is Option E.

A contraindication for web correction is super digits. As these digits grow, common complications include enlargement and undergrowth; therefore, reconstruction into normally functioning digits is rare.

Other contraindications to surgery involve complex syndactyly with conjoined fingers functionally moving in unison; separating these jumbled masses may significantly jeopardize postoperative function. Surgery also is contraindicated in hands with no active muscular control.

In constriction band syndrome, syndactyly may be mild or severe. In Poland syndrome, syndactylies with hypoplasia are found in variations. In Apert syndrome, syndactyly patterns include complex syndactyly of the index, long, and ring fingers, and simple syndactyly of the web space between the ring and little fingers.

48
Q

Which of the following factors is NOT associated with an increased risk of developing the condition shown in the photograph?
A ) Family history
B ) Low birth weight (less than 2500 g)
C ) Multigravida mother
D ) Prematurity (less than 37 weeks
E ) Young maternal age (less than 25 years)

A

The correct response is Option A.

Amniotic band syndrome is a sporadic condition in which rupture of the amnion results in strands of tissue that ensnare the developing fetus. The factors associated with this condition include prematurity, low birth weight, and young multigravida mothers. This condition is sporadic and mechanical in nature. As such, it does not have a genetic basis or identifiable pattern of inheritance.

49
Q

A newborn is brought to the office because of the deformity shown in the photograph. Which of the following findings is most likely associated with this condition?

A ) Autosomal recessive inheritance

B ) High plasminogen levels

C ) Infection during the second trimester

D ) Injury to the subclavian artery

E ) Oligohydramnios

A

The correct response is Option E.

The patient described is brought to the office with the classic findings of skin banding and acrosyndactyly consistent with amniotic band syndrome (ABS). The incidence of ABS has been reported at 1:15,000 births with equal expression in males and females. No autosomal inheritance pattern has been identified nor has a connection been made to any infectious agent. An association with oligohydramnios may explain the increased frequency of clubfoot deformity; however, this finding is not ubiquitous. Researchers have postulated that an early transient oligohydramnios at the time of amniotic disruption is most likely causative.

Anomalies such as calcaneovalgus, pes valgo planus, congenital hip dislocation, and congenital vertical talus can also be associated with oligohydramnios.

Vasculocutaneous catastrophe of the newborn is associated with an altered fibrinolytic system, including high plasminogen levels and antiplasmin inhibitors. Vascular injury to the subclavian artery has been a suspected cause of symbrachydactyly but not ABS.

50
Q

A newborn child is brought to the office because of the hand deformity shown in the photograph and the radiograph. Which of the following is the most appropriate classification for this deformity?

A ) Duplication
B ) Failure of differentiation
C ) Failure of formation: longitudinal
D ) Failure of formation: transverse
E ) Overgrowth

A

The correct response is Option B.

Approximately one in 600 infants is born with a congenital difference of the upper limb. The American Society of Surgery of the Hand and International Federation of Societies for Surgery of the Hand developed a unified classification system to help reduce the confusion from the previous system of Greek and Latin prefixes.

Classification of Congenital Hand Differences Type/Description

I Failure of formation of parts IA Transverse (eg, amelia, more distal amputations) IB Longitudinal (eg, phocomelia, radial dysplasia) II Failure of differentiation (separation) of parts (eg, synostosis, syndactyly, contracture,

camptodactyly, trigger digits, clinodactyly) III Duplication (eg, polydactyly, triphalangeal thumb) IV Overgrowth (eg, macrodactyly) V Undergrowth (eg, hypoplastic thumb) VI Constriction band syndrome VII Generalized skeletal deformities (eg, dystrophic dwarfism)

The photograph shows the hand of a newborn child with a simple syndactyly, which is a conjoining of the adjacent fingers. This syndactyly would be classified as a failure of differentiation. Surgical treatment would be performed in a staged procedure, avoiding separating adjacent web spaces for fear of devascularizing a finger. Separation is usually delayed until the child is one year old; however, earlier separation may be needed to separate the small and ring finger syndactyly, if there is a significant difference in the digital length. Surgical repair of the deformity involves multiple skin flaps from the adjacent fingers, along with full-thickness skin grafting. Releases should be performed before the child is of school age.

51
Q

A 2-month-old infant is scheduled to undergo staged surgical release of complete syndactyly of every web space. Which of the following web spaces are most appropriately addressed during the first stage of syndactyly release?

A ) 1st and 2nd

B ) 1st and 4th

C ) 2nd and 3rd

D ) 2nd and 4th

E ) 3rd and 4th

A

The correct response is Option B.

Syndactyly is a condition resulting from failure of the normal separation process during five to eight weeks of gestation. It occurs in approximately one in 2000 births, either spontaneously or as part of a syndrome. Syndactyly is classified as complete or incomplete based on degree of distal to proximal webbing. It is also described as complex or simple, reflecting the presence or absence of bony involvement.

When syndactyly involves multiple digits, a plan must be developed to accomplish as much as safely possible at each operation to minimize the total number of operations. Border digit syndactyly should be addressed between four and six months to minimize tethering effects and abnormal growth. Complete syndactyly of adjacent web spaces should not be performed at the same stage due to the increased risk of vascular compromise to the digit.

In complete syndactyly, the border digits (1st and 4th web spaces) are usually released first. The 2nd and 3rd web spaces are released later in two separate operations. If there is no border digit tethering, complete release can be performed in two operations by first releasing the 1st and 3rd web spaces, followed by the 2nd and 4th web spaces.

52
Q

A male newborn has the isolated congenital deformities of the left hand shown in the photographs. Which of the following procedures is most likely to provide optimal function of the hand?

(A) Amputation of the long, ring, and small fingers

(B) Full-thickness matrix grafting

(C) Phalangeal lengthening of the small and ring fingers

(D) Radial metacarpal lengthening of the index and long fingers

(E) Toe-to-hand transplantation

A

The correct response is Option C.

The patient described has an isolated congenital anomaly of the left hand known as shortened fingers or brachydactyly. The most functional improvement is to lengthen the small and ring fingers to facilitate power grip and opposition. One method is nonvascularized toe phalanx grafting; another is distraction lengthening. This procedure should be performed when the patient is between 8 and 12 months of age.

Amputation would not add to the patient €™s function. Full-thickness matrix grafting is for reconstruction of the nail matrix complex. Lengthening the second and third metacarpals would not address the functional deficit. The patient €™s thumb appears to be quite functional, so toe-to-hand transplantation is not appropriate.

53
Q

A 6 year old boy is brought to the office because of enlargement of the index and long fingers that has been progressing since infancy. A photograph of the fingers is shown. He has not had pain. Two €‘point discrimination is 6 to 8 mm in the affected digits. Which of the following is the most appropriate treatment?

(A) Administration of thyroid supplements

(B) Radiation therapy to the fingers

(C) Ray amputation of the index finger

(D) Surgical debulking and epiphysiodesis

(E) Systemic administration of oral corticosteroids

A

The correct response is Option D.

The photograph shows a case of macrodactyly of both the index and long fingers. This deformity is caused by the development of a lipomatous hamartoma usually within the digital nerves with resulting overgrowth of the hand and fingers within the distribution of the tumor. If the affected fingers are sensate, then debulking procedures, in addition to epiphysiodesis (premature closure of the growth plates), at the distal interphalangeal, proximal interphalangeal, and metacarpophalangeal joints may slow progression of the disease process. Epiphysiodesis should be performed when the fingers are at the adult length.

Radiation and thyroid supplementation are not known to affect the growth of such tumors and carry significant side effects. The affected digits are often insensate and stiff with cold intolerance. When indicated by painful, functionally massive overgrowth, amputation is an option, although generally not initially. Systemic administration of corticosteroids is the treatment for some forms of congenital hemangiomas.

54
Q

A 7-year-old boy is brought to the emergency department one hour after sustaining a fracture of the small finger of the nondominant left hand when he fell down a flight of stairs. A radiograph of the left hand is shown. Which of the following is the most likely diagnosis?

(A) Extra €‘octave fracture

(B) Fracture-dislocation

(C) Salter €‘Harris type I fracture

(D) Salter €‘Harris type II fracture

(E) Salter €‘Harris type III fracture

A

The correct response is Option A.

This is an extra €‘octave fracture that does not involve the physis. The fracture line is very close to the physis but is above it. Forty-one percent of pediatric hand fractures involve the physis. Most involve the phalanges, particularly the proximal phalanx. The reason for this high rate is that the collateral ligaments insert into the epiphysis.

All Salter-Harris-type fractures involve the physis. Traditionally, pediatric physeal fractures are described using the Salter-Harris classification system. There are five types of Salter-Harris fractures (see below). Type I fractures are transphyseal. Type II are transphyseal but exit through the metaphysis. Type III are transphyseal fractures that exit the epiphysis and the joint. Type IV fractures traverse both the epiphysis and physis, exiting through the metaphysis. Type V fractures describe crush injuries to the physis.

The joint has not been affected in the fracture described and would therefore not be considered a fracture €‘dislocation. If it had involved the physis, this extra €‘octave fracture would likely have been a Salter €‘Harris Type II fracture, the most common type of fracture in this population.

55
Q

A 7-year-old girl is brought to the office by her parents because she has a deformity of the left hand. Medical history is unknown because the patient was recently adopted. She is otherwise healthy. Examination of the right hand shows no abnormalities. Her parents say that she is able to play with her toys with both hands and can pick up small objects with the left hand. A photograph of the hands is shown. Which of the following is the most appropriate option for surgical reconstruction?

(A) Distraction lengthening of the thumb

(B) Microvascular transfer of second and third toes to the index and long finger positions

(C) Microvascular transfer of second and third toes to the small and ring finger positions

(D) Transfer of toe phalangeal epiphysis to the index and long finger positions

(E) No surgical reconstruction

A

The correct response is Option E.

Congenital hand deformities present a challenge to the patient, parents, and the reconstructive surgeon. Transverse arrest or congenital amputations may present as a failure of formation or arrest of development of the limb. Operative indications for this group of patients are often limited but are driven by the function of the child. The patient described is a very functional child who uses her limb for complex grasping activities; therefore, no surgery is indicated.

Distraction lengthening of the thumb is not appropriate for the patient described since it may impede pulp-to-pulp opposition of the radial and ulnar digits. Microvascular transfer of one or two toes may be a worthwhile procedure in children with no opposable digits and no functional use of their hand. However, in the patient described, this procedure is not appropriate because function is already satisfactory. Toe epiphysis nonvascular transfers have been indicated in congenitally short digits, but for the patient described there are no normal phalanges to warrant this procedure.

56
Q

A 1-year-old boy is brought to the office by his parents for consultation of a flexion deformity of the right thumb. Physical examination shows fixed flexion of the thumb at the interphalangeal joint. No other abnormalities are noted. Which of the following is the most likely diagnosis?
(A) Camptodactyly
(B) Clinodactyly
(C) Delta phalanx
(D) Macrodactyly
(E) Trigger thumb

A

The correct response is Option E.

Trigger digit reportedly occurs with an incidence of 1 in 2000 to 3 in 1000 births. Whereas most authorities believe this deformity is a congenital abnormality, there is debate whether trigger digit is an acquired deformity. The congenital trigger finger presents as a flexed digit either at birth or up to 3 years of age. Often it may not be recognized early because of the tendencies of infants to hold their fingers in a flexed position. Before 12 months of age, observation is the standard treatment. After 3 years of age, surgical release is generally recommended. The results of conservative nonoperative treatment and spontaneous recovery have been reported as between 0% and 30%.

Trigger digit can be confused with the diagnosis of camptodactyly, which is a congenital flexion posture most often found in the small finger. Although congenital trigger thumb is related to a nodule in the tendon, the etiology of camptodactyly is less clear, with various etiologies related to skin deficit, volar plate abnormalities, and abnormalities of the lumbrical and superficialis tendons.

Abnormally angulated digits in the radial ulnar plane is clinodactyly. Clinodactyly may be associated with a delta phalanx, which is an abnormally shaped, short tubular bone within the digit.

Macrodactyly refers to an abnormal enlargement of the digit. In true macrodactyly, all structures of the digits are enlarged.

57
Q

A 12-year-old boy is brought to the emergency department because he has had episodes of syncope for the past week. Physical examination of the hands shows hypoplasia of the left thumb. Electrocardiography shows atrial fibrillation. Which of the following is the most likely diagnosis?
(A) Holt-Oram syndrome
(B) Fanconi anemia
(C) Robert syndrome
(D) TAR syndrome
(E) VATER association

A

The correct response is Option A.

Holt-Oram syndrome is an autosomal-dominant disorder involving cardiac defects and upper limb anomalies. Common cardiac defects include atrial or ventricular septal defects and conduction abnormalities or blocks. Upper limb anomalies are variable in type and severity, ranging from subtle pre-axial carpal abnormalities to absent thumbs or radial club hand. The disorder is due to a mutation in a gene encoding a transcription factor involved in both cardiac and upper limb development.

Fanconi anemia is primarily characterized by bone marrow failure. It is an autosomal-recessive disease due to mutations in a series of genes believed to be involved in DNA repair. Many patients are born with a variety of radial limb anomalies, including hypoplastic thumbs or absent radii. Other common anomalies include café au lait spots, short stature, genital abnormalities, and microcephaly. Patients usually die of complications from pancytopenia, with a life expectancy of 20-30 years.

TAR syndrome involves thrombocytopenia and absent radii. It can be distinguished from Fanconi anemia by the neonatal onset of thrombocytopenia, which is typically seen later in life with Fanconi anemia, and the absence of pancytopenia.

Robert syndrome is a very rare disorder characterized by tetraphocomelia, similar to that seen with thalidomide embryopathy, as well as brain and craniofacial abnormalities.

VATER is an acronym for vertebral/vascular anomalies, anal atresia, tracheo-esophageal fistula, esophageal atresia, and renal/radial anomalies. Cardiac defects, other limb anomalies, and a single umbilical artery are also associated with this complex, giving rise to the alternative acronym VACTERLS.

58
Q

Hematologic evaluation is indicated before surgical intervention for which of the following conditions?

(A) Atypical cleft hand

(B) Constriction band syndrome

(C) Radial hypoplasia

(D) Small finger polydactyly

(E) Thumb duplication

A

The correct response is Option C.

Radial aplasia and hypoplasia are associated with several syndromes including VACTERL (vertebral, anal, cardiac, tracheoesophageal, renal/radial, limb), Holt €‘Oram, TAR (thrombocytopenia €‘absent radius), and Fanconi anemia. Both TAR syndrome and Fanconi anemia are contraindications for early surgical intervention until the underlying hematologic abnormalities have stabilized or been corrected.

The other congenital disorders listed tend to occur sporadically or familially and are not associated with specific hematologic abnormalities.

59
Q

A 3-month-old infant is brought to the office for consultation regarding deformity of the right hand (shown). The infant is otherwise healthy and has no other abnormalities. On physical examination, no abnormalities other than those of the right hand are noted. Radiographs show no bony syndactyly. The most appropriate management of this patient €™s deformity is release of the syndactyly and which of the following?

(A) Amputation of the digits

(B) First dorsal interosseus flap

(C) Full-thickness skin grafting

(D) Radial artery flap

(E) Split-thickness skin grafting

A

The correct response is Option C.

Syndactyly is the second most common congenital hand abnormality after polydactyly, occurring in 1 in 2000 live births. Syndactyly can be described as complete, when the interdigital web extends to the full length of the digit, or incomplete. Although there have been reports of the use of skin flaps or primary closure after radical subcutaneous tissue debulking, skin grafts are almost always necessary. Full-thickness skin grafting is the appropriate management of syndactyly in a child. Effective grafts for this procedure include the lateral groin crease, plantar foot, antecubital fossa, inner arm, or wrist flexion crease.

Amputation of the digits is almost never required for a simple syndactyly.

Pedicle flaps such as a dorsal interosseus flap or radial artery flap are not required because tissue can be closed with adjacent flaps and skin grafts alone.

A split-thickness skin graft is a poor choice in a young child because it is difficult to obtain and has a significant amount of contracture.

60
Q

A 2-year-old boy is referred to the office by a pediatrician for surgical correction of a deformity of the right hand. Physical examination shows Wassel Type 4 duplication of the thumb. In surgical management of this patient €™s condition, which of the following is the most critical point?

(A) Amputation of the ulnar duplication

(B) Maintenance of the ulnar collateral ligament at the metacarpophalangeal joint

(C) Reattachment of the abductor pollicis brevis

(D) Repair of the flexor pollicis longus tendon

(E) Repair of the nail bed

A

The correct response is Option B.

There are seven classes of thumb duplication as described by Wassel. Type 4 is the most common. Type 1 involves a duplication of the distal phalanx in which the two are connected at the base. Type 2 duplications occur when the two distal phalanges are separate. Type 3 duplications are of the proximal phalanx and are connected at the base. Type 4 duplications are of the distal and proximal phalanxes when there are four distal and proximal phalanges. Type 5 are duplications of the distal and proximal phalanxes with a duplication of the metacarpal still fused at the base. Type 6 duplications result in six separate bones, two distal phalanges, two proximal phalanges, and two metacarpals that are separate.

In the correction of Type 4 duplications, it is most critical to preserve the ulnar collateral ligament to stabilize pinch. Typically, the radial thumb is less functional and less developed. The radial collateral ligament (RCL) is reconstructed with a ligament-periosteal flap. The abductor pollicis brevis is included in this RCL reconstruction. Most often, the flexor pollicis longus in the ulnar element is intact and does not require repair or reconstruction. As seen in the photograph of the thumb duplication in the patient described, the nail complex does not require intervention.

61
Q

A 1-year-old boy is brought to the office by his parents for consultation of a flexion deformity of the right thumb. Physical examination shows fixed flexion of the thumb at the interphalangeal joint. No other abnormalities are noted. Which of the following is the most likely diagnosis?
(A) Camptodactyly
(B) Clinodactyly
(C) Delta phalanx
(D) Macrodactyly
(E) Trigger thumb

A

The correct response is Option E.

Trigger digit reportedly occurs with an incidence of 1 in 2000 to 3 in 1000 births. Whereas most authorities believe this deformity is a congenital abnormality, there is debate whether trigger digit is an acquired deformity. The congenital trigger finger presents as a flexed digit either at birth or up to 3 years of age. Often it may not be recognized early because of the tendencies of infants to hold their fingers in a flexed position. Before 12 months of age, observation is the standard treatment. After 3 years of age, surgical release is generally recommended. The results of conservative nonoperative treatment and spontaneous recovery have been reported as between 0% and 30%.

Trigger digit can be confused with the diagnosis of camptodactyly, which is a congenital flexion posture most often found in the small finger. Although congenital trigger thumb is related to a nodule in the tendon, the etiology of camptodactyly is less clear, with various etiologies related to skin deficit, volar plate abnormalities, and abnormalities of the lumbrical and superficialis tendons.

Abnormally angulated digits in the radial ulnar plane is clinodactyly. Clinodactyly may be associated with a delta phalanx, which is an abnormally shaped, short tubular bone within the digit.

Macrodactyly refers to an abnormal enlargement of the digit. In true macrodactyly, all structures of the digits are enlarged.

62
Q

A 12-year-old boy is brought to the emergency department because he has had episodes of syncope for the past week. Physical examination of the hands shows hypoplasia of the left thumb. Electrocardiography shows atrial fibrillation. Which of the following is the most likely diagnosis?
(A) Holt-Oram syndrome
(B) Fanconi anemia
(C) Robert syndrome
(D) TAR syndrome
(E) VATER association

A

The correct response is Option A.

Holt-Oram syndrome is an autosomal-dominant disorder involving cardiac defects and upper limb anomalies. Common cardiac defects include atrial or ventricular septal defects and conduction abnormalities or blocks. Upper limb anomalies are variable in type and severity, ranging from subtle pre-axial carpal abnormalities to absent thumbs or radial club hand. The disorder is due to a mutation in a gene encoding a transcription factor involved in both cardiac and upper limb development.

Fanconi anemia is primarily characterized by bone marrow failure. It is an autosomal-recessive disease due to mutations in a series of genes believed to be involved in DNA repair. Many patients are born with a variety of radial limb anomalies, including hypoplastic thumbs or absent radii. Other common anomalies include café au lait spots, short stature, genital abnormalities, and microcephaly. Patients usually die of complications from pancytopenia, with a life expectancy of 20-30 years.

TAR syndrome involves thrombocytopenia and absent radii. It can be distinguished from Fanconi anemia by the neonatal onset of thrombocytopenia, which is typically seen later in life with Fanconi anemia, and the absence of pancytopenia.

Robert syndrome is a very rare disorder characterized by tetraphocomelia, similar to that seen with thalidomide embryopathy, as well as brain and craniofacial abnormalities.

VATER is an acronym for vertebral/vascular anomalies, anal atresia, tracheo-esophageal fistula, esophageal atresia, and renal/radial anomalies. Cardiac defects, other limb anomalies, and a single umbilical artery are also associated with this complex, giving rise to the alternative acronym VACTERLS.

63
Q

In a child with Type III B hypoplastic thumb, the most appropriate thumb reconstruction is

(A) ablation and index finger pollicization
(B) amputation
(C) basilar joint reconstruction
(D) distraction osteogenesis lengthening
(E) Huber transfer

A

The correct response is Option A.

The five categories in the classification of congenital hypoplastic thumb range from Type I, for a small thumb with all essential components, to Type V, for complete absence of the thumb. Type III is subdivided into type A, for intact carpometacarpal joint, and type B, for absence of the joint.

Because of the instability associated with absence of the carpometacarpal joint, the most appropriate reconstruction of Type III B deformity is ablation of the thumb and pollicization of the index finger.
The goal of hypoplastic thumb reconstruction is a functional thumb. Amputation, distraction lengthening, and basilar joint reconstruction do not result in functional thumb. Furthermore, adequate autologous elements are not present for basilar joint reconstruction.

A Huber transfer is used to create functional opposition when absent. Type III A deformities can be reconstructed with the Huber transfer, which involves transfer of the hypothenar muscle to recreate the thenar musculature.

64
Q

During reconstruction of the split thumb deformity shown above, transfer of which of the following muscles is most effective for long-term skeletal stability of the thumb?

(A) Abductor pollicis brevis
(B) Adductor pollicis
(C) Extensor carpi radialis longus
(D) Extensor indicis proprius
(E) Flexor digitorum superficialis ring

A

The correct response is Option A.

This patient has a Wassel Type IV split thumb deformity. Surgical reconstruction of this deformity involves transfer of the abductor pollicis brevis muscle from the radial base of the radial duplicate to the radial base of the retained ulnar duplicate. This muscle transfer is crucial to balance the force of the adductor pollicis brevis, which is attached to the proximal ulnar aspect of the proximal phalanx. If the abductor pollicis brevis is not transferred, the thumb will eventually collapse into a Z-deformity, with ulnar deviation of the proximal phalanx and radial deviation of the distal phalanx. Normally, the abductor pollicis brevis tendon is removed from the proximal phalanx with a cuff of periosteum, which facilitates attachment to the radial aspect of the proximal phalanx of the ulnar duplicate. This transfer is protected by a longitudinal Kirschner wire placed through the metacarpophalangeal joint. The thumb is then immobilized in a long arm cast for four weeks to protect the surgical repair.

Reconstruction of any duplicate thumb deformity always requires an inventory of all tendon and muscle units. In general, the ulnar duplicate is retained because of the stability of the ulnar collateral ligament of the metacarpophalangeal joint. Augmentation of the extrinsic flexor or extensor tendons is performed occasionally using the tendon and muscle units that would have been discarded after deletion of the radial duplicate. In addition, these deformities should be examined for eccentrically inserted extrinsic tendons, which must be centralized to ensure skeletal stability.

The adductor pollicis is firmly inserted into the ulnar base of the proximal phalanx of the retained ulnar duplicate and is not transferred. The extensor carpi radialis longus tendon, extensor indicis proprius tendon, and flexor digitorum superficialis tendon to the ring finger are useful in reconstruction of palsy of the median and ulnar nerves but have no role in reconstruction of duplicate thumb deformity.

65
Q

A 7-year-old girl is brought to the office by her parents for consultation regarding surgical correction of camptodactyly of the small fingers. Which of the following interventions is the most appropriate management of this deformity?

(A) Observation only
(B) Corrective osteotomy of the proximal phalanges
(C) Lengthening of the flexor tendons
(D) Release of the volar plate
(E) Tightening of the extensor mechanism

A

The correct response is Option A.

Camptodactyly is a congenital flexion deformity of the proximal interphalangeal joints of the small fingers. It is usually not a functional problem because patients with the condition make accommodations to adapt to the deformity. Surgical repair, including corrective osteotomy, lengthening of the flexor tendons, release of the volar plate, and tightening of the extensor mechanism, is usually unsatisfactory and may cause stiffness of the small finger. Although the parents of this patient may try to persuade the surgeon to correct the deformity, the appropriate response for the surgeon is to resist their pressure and reassure them that this is not a functional concern. Surgical correction should be the last possible option.

66
Q

A neonate has complete absence of the left thumb. Which of the following is the most likely associated finding?

(A) Absent pectoralis major muscle
(B) Arthrogryposis
(C) Hematopoietic abnormalities
(D) Scleroderma
(E) Trisomy 19

A

The correct response is Option C.

Congenitally absent thumb syndrome has no known cause but may be a genetic anomaly with sporadic occurrence. Other environmental factors, such as viruses, drugs, or exposure to radiation or chemicals, may also cause this condition.

Blauth’s classification of congenital absence of the thumb is explained below.

Type I: Shortened thumb, normal bones and muscles
Type II: Shortened thumb, hypoplasia of the thenar muscles and possible contracture of the first web space, narrowed bones, associated instability of the ulnar collateral ligament
Type III: Shortened thumb, absence of the thenar muscles, resulting in severe adduction contracture, instability of the metacarpophalangeal joint, variable abnormalities of the metacarpal, scaphoid, trapezium, and radial styloid
Type IV: Pouce flottant (floating thumb), absence of the thenar muscles
Type V: Complete absence of the thumb
Conditions associated with congenital absence of the thumb include:

  1. Spinal abnormalities (VATER [vertebral, anal, tracheoesophageal, radial and renal] syndrome)
  2. Cardiovascular anomalies (Holt-Oram syndrome)
  3. Hematopoietic anomalies (Fanconi anemia, thrombocytopenia–absent radius [TAR] syndrome)
  4. Renal abnormalities (VATER syndrome)
  5. Gastrointestinal anomalies
  6. Cornelia de Lange syndrome
  7. Hand-foot-uterus syndrome
  8. Trisomy 18
  9. Brachydactyly
  10. Rubinstein-Taybi syndrome
  11. Apert syndrome
  12. Carpenter syndrome
  13. Myositis ossificans
67
Q

For each patient who has a longitudinal deficiency of the radius, select the most likely diagnosis.

(A) Fanconi anemia
(B) Holt-Oram syndrome
(C) Nager syndrome
(D) TAR syndrome
(E) VATER syndrome

198 A 2-month-old girl has a decreased platelet count

199 A 7-year-old boy has progressive pancytopenia

200 A 6-month-old girl has defects of the atrial and ventricular septa

A

The correct response for Item 198 is Option D, for Item 199 is Option A, and for Item 200 is Option B.

The 2-month-old girl with radial longitudinal deficiency and decreased platelet count most likely has TAR (thrombocytopenia–absent radius) syndrome, an autosomal recessive disorder. The low platelet count, which is detected during the neonatal period, is most likely to improve over time, and the prognosis for patients with TAR syndrome is good.

The 7-year-old boy with radial longitudinal deficiency and progressive pancytopenia has Fanconi anemia, an autosomal dominant condition that may not appear until mid-childhood. Mitomycin C testing is recommended to confirm the diagnosis. The long-term prognosis for patients with Fanconi anemia is poor.

The 6-month-old girl with radial longitudinal deficiency and defects of the atrial and ventricular septa most likely has Holt-Oram syndrome, which is autosomal dominant. Clavicular hypoplasia is also characteristic.

Nager syndrome is an autosomal recessively inherited condition that manifests as radial deficiency and craniofacial abnormalities.

VATER syndrome is characterized by vertebral anomalies, anal atresia or imperforation, tracheoesophageal atresia, and renal and radial defects. Some patients may also have lower limb abnormalities.

68
Q

A 6-month-old boy has the deformities shown in the photograph above. There are no associated syndromes. His parents would like to have more children and inquire about the risk for similar deformities. The risk for development of this deformity in future offspring is closest to

(A) 0%
(B) 5%
(C) 10%
(D) 25%
(E) 50%

A

The correct response is Option A.

This 6-month-old boy has amniotic constriction band syndrome, a congenital hand deformity that manifests as intrauterine amputation of the digits. Amniotic constriction band syndrome has no known genetic transmission and is considered to be an intrauterine accident. Although there are several theories regarding the cause of this condition, it is believed to be related to rupture of the amniotic membrane caused by oligohydramnios; when this occurs, the digits and/or extremities are constricted by amniotic tissue bands created from the edges of the ruptured amniotic sac. Amniotic bands may result in amputations, not only of the fingers (as in this patient) or the extremities, but sometimes other parts of the body. The degree of constriction is variable and is classified according to its severity.

Amniotic constriction band syndrome is the fourth most common congenital anomaly of the hand; only polydactyly, syndactyly, and trigger thumb occur more frequently. Currently, its incidence is estimated at approximately 1 in 5000 to 15,000 births, with variations among different ethnic groups.

Anomalies frequently associated with amniotic constriction band syndrome include club foot, cleft lip and/or palate, craniofacial defects, hemangioma, and meningocele.

In this infant, appropriate management of the hand deformity is deepening of the web space using a standard dorsal flap followed by full-thickness skin grafting.

69
Q

During embryologic development of the hand, which of the following structures is most likely to affect normal differentiation of the limb?

(A) Apical ectodermal ridge
(B) Fourth branchial cyst
(C) Germinal matrix
(D) Second branchial cleft
(E) Wolffian ridge

A

The correct response is Option A.

The apical ectodermal ridge is the critical structure that defines the growth and differentiation of the new limb during embryologic development; if the apical ectodermal ridge were to be surgically removed, the developing limb would be truncated. The apical ectodermal ridge arises from the Wolffian ridge, which protrudes from the main trunk of the embryo. A zone of polarizing activity determines the anterior-posterior morphology of the limb.

The upper extremities begin to develop during the first four weeks of gestation. By the completion of the fifth week, the hand becomes a recognizable structure. As the apical ectodermal ridge becomes flattened, the hand appears initially as a paddle. Each of the phalanges is then formed by a process of physiologic cell death that occurs within the web spaces. The digits become defined by the end of the eighth week, and fingernails can be identified by 17 weeks’ gestation.

70
Q

A 6-year-old girl has camptodactyly of the small finger of the right hand. The deformity has not improved with splinting and passive stretching exercises. On physical examination, there is loss of 20 degrees of terminal extension of the proximal interphalangeal (PIP) joint that is unaffected by flexion of the metacarpophalangeal joint.

Which of the following is the most appropriate management?

(A) Observation
(B) Injection of a corticosteroid into the PIP joint
(C) Release of the lumbrical tendon
(D) Release of the superficialis tendon
(E) Zancolli-lasso procedure

A

The correct response is Option A.

In this 6-year-old girl with camptodactyly who has a loss of 20 degrees of extension of the proximal interphalangeal (PIP) joint, observation is most appropriate. Surgical correction is rarely required in patients who have lost less than 30 degrees of extension. Minimal extension deficits that require operative management can be corrected by transferring the lumbrical tendon into the central slip.

In patients who have a terminal extension deficit of 30 degrees or more, surgical release of the abnormal lumbrical and superficialis tendons is most likely to improve the deformity. Release of the attachment of the accessory collateral ligaments to the volar plate may also be necessary. Full-thickness skin grafts or local skin flaps can be used to cover the resultant volar skin deficits. However, the patient should be informed of the likelihood for a residual flexion deformity, especially if there are abnormalities of the articular joint surfaces on preoperative radiographs.

Corticosteroids should not be injected into the affected joints of patients with camptodactyly. In the Zancolli-lasso procedure, slips of the flexor digitorum superficialis tendon are looped through the A2 pulley. This technique is appropriate for correction of digital clawing associated with ulnar palsy in a patient who demonstrates improved extension of the PIP joint with flexion of the metacarpophalangeal joint.

71
Q

A 10-month-old infant with simple, complete syndactyly involving the third web space of the left hand undergoes reconstruction using a dorsal flap and multiple full-thickness skin grafts. Which of the following is the most appropriate next step in management?

(A) Wound cleansing using dilute peroxide and application of antibiotic ointment daily
(B) Application of soft dressings and an elastic wrap
(C) Application of a short arm splint
(D) Application of a short arm cast
(E) Application of a long arm cast

A

The correct response is Option E.

In infants and children who undergo reconstructive procedures for correction of syndactyly, the most appropriate management postoperatively is application of a long arm cast. The success of the procedure depends on adequate immobilization of the entire upper extremity following surgery. When applying the cast, the elbow must be flexed before padding and fiberglass material are applied in order to prevent restriction or pressure in the antecubital region. Flexion of the cast at the elbow will prevent the dressings around the skin grafts and flaps from becoming loosened.

The photograph above shows the properly applied above-elbow cast of a patient who has undergone syndactyly reconstruction. All of the fingertips are visible, but the fingers are immobilized securely. This infant previously underwent two reconstructive attempts that were unsuccessful because of improper postoperative immobilization. The third procedure, involving release of severe contractures of the digits followed by repeat full-thickness skin grafting, was successful because the arm was immobilized correctly.

If the arm is not immobilized, the skin grafts and flaps will ultimately fail, and significant wound problems will occur. Soft dressings do not provide the needed immobilization. Short arm splints and casts frequently become dislodged in infants in children because of their high level of activity and the problems associated with fitting a short arm device to the cone-shaped forearm of a young child.

72
Q

A 3-year-old boy has total absence of the long and ring fingers of both hands. This finding is most consistent with which of the following conditions?

(A) Camptodactyly
(B) Clinodactyly
(C) Ectrodactyly
(D) Macrodactyly
(E) Polysyndactyly

A

The correct response is Option C.

This 3-year-old boy has ectrodactyly, a partial or total absence of the fingers that suggests a central hand deficiency. Central hand deficiencies can be classified as typical or atypical. Patients with typical central hand deficiencies have absence of the third ray (infrequently the second ray, rarely the fourth ray). The finding is often bilateral. Other anomalies, including cleft lip and palate, congenital heart disease, and significant deformities of the upper and lower extremities, can be associated. Syndactyly and foot involvement are common. The inheritance is familial.

In contrast, a patient with atypical central hand deficiency usually has several deficient rays unilaterally. This deficiency is commonly known as symbrachydactyly and is the opposite of true cleft hand. Inheritance is nonfamilial. Other anomalies, such as syndactyly and foot involvement, are uncommon. Affected patients typically have digital nubbins that are best removed at birth. Functional hand grasp can be increased through surgical manipulation of the web space or osteotomies.

Camptodactyly is a nontraumatic flexion deformity of the proximal interphalangeal joint, usually bilateral and involving the small finger.

Patients with clinodactyly have either radial or ulnar angulation of the digit, usually the small finger, at the distal interphalangeal joint. In severe clinodactyly, a delta phalanx is common.

Macrodactyly involves overgrowth of one digit or hand segment. Overgrowth appears to be under neurotrophic control. Correction is either reduction or ablation.

Polysyndactyly is a type of complicated syndactyly, in which polydactyly and syndactyly are seen in association.

73
Q

A 9-month-old infant has the deformity shown in the photograph above. The pregnancy and delivery were uncomplicated; the infant’s development is otherwise normal. Which of the following is the most likely diagnosis?
(A) Brachysyndactyly
(B) Camptodactyly
(C) Clinodactyly
(D) Ectrodactyly
(E) Syndactyly

A

The correct response is Option E.

Syndactyly is a common congenital hand anomaly, occurring in one of every 2500 neonates. Familial inheritance ranges from 10% to 40%, and boys are twice as likely as girls to be affected. Syndactyly can be simple or complex and complete or incomplete. Patients with simple syndactyly have fusion of the skin of adjacent digits. In complex syndactyly, the skin and bones are fused. Complete syndactyly encompasses the entire web space. In patients with incomplete syndactyly, only part of the web space is fused.

Based on the clinical photograph alone, incomplete syndactyly can be diagnosed in this infant because the entire web space is not involved. However, because there are no additional radiographs, a diagnosis of simple or complex syndactyly cannot be determined.

Although most infants with syndactyly undergo release before 18 months of age, some surgeons have performed surgery as early as 6 weeks of age.

Infants with brachydactyly have short, fused digits. Camptodactyly is characterized by curvature of a digit in its plane of flexion; in contrast, clinodactyly describes deviation of the digit in the plane of the hand. Patients with ectrodactyly have partial or total absence of the digits.

74
Q

A 7-month-old infant has the deformity shown in the photographs above. Which of the following is the most likely mode of inheritance of this deformity?

(A) Autosomal dominant
(B) Autosomal recessive
(C) Sporadic
(D) X-linked dominant
(E) X-linked recessive

A

The correct response is Option A.

This 7-month-old infant has a typical, or true, cleft hand deformity, which is most commonly inherited as an autosomal dominant condition. However, cleft hand may also occur as a spontaneous mutation, as it did in this patient whose family members were unaffected.

Typical cleft hand deformity is classified as a longitudinal deficiency because it involves dysplasia of the central portion of the hand. This anomaly is stratified into five categories, based on the degree of ray absence and hypoplasia of the thumb space. Characteristics include a V-shaped cleft, suppression of the central digit, and sometimes minor syndactyly of the ulnar border digits.

In contrast, atypical cleft hand is sporadic and is included in the teratologic sequence of symbrachydactyly. Affected patients have a broad, flat cleft hand with finger nubbins containing nail remnants.

Because typical cleft hand may be associated with several syndromes, any infant who exhibits this anomaly should be evaluated for cardiac, pulmonary, musculoskeletal, ocular, and renal defects. EEC syndrome, involving ectrodactyly, ectodermal dysplasia, and cleft lip/palate, is most frequently associated. Many patients with typical cleft hand deformity also have clefting of the feet (as does this patient), which results from localization of the split hand foot gene (SHFM1) to chromosome 7q21.

The degree of deformity of typical cleft hand varies greatly. Although most patients adapt and ultimately function quite well, surgical reconstruction is often recommended for functional as well as cosmetic reasons. The Snow-Littler procedure, which involves transposition of a palmar-based cleft flap with the index ray to close the cleft and create a useful first web space, is an appropriate reconstructive option.

75
Q

A 9-month-old infant has the deformity shown in the photograph above. Which of the following is the most appropriate age for correction of the deformity?

(A) Birth to 3 months
(B) 4 to 9 months
(C) 12 to 18 months
(D) 24 to 30 months
(E) After 36 months

A

The correct response is Option C.

Syndactyly is a common congenital hand anomaly, occurring in one of every 2500 neonates. Familial inheritance ranges from 10% to 40%, and boys are twice as likely as girls to be affected. Syndactyly can be simple or complex and complete or incomplete. Patients with simple syndactyly have fusion of the skin of adjacent digits. In complex syndactyly, the skin and bones are fused. Complete syndactyly encompasses the entire web space. In patients with incomplete syndactyly, only part of the web space is fused.

The infant shown in the photograph above has incomplete syndactyly because the entire web space is not involved. However, because there are no additional radiographs, a diagnosis of simple or complex syndactyly cannot be determined.

Most infants with syndactyly undergo release before 18 months of age. During this period, the digits have grown to an adequate size, anesthetic complications are decreased, and hand dominance has not yet developed. Some surgeons have performed surgery as early as 6 weeks of age. Infants who have syndactyly of the other web spaces, especially the first and fourth, should undergo surgery at an earlier age because of the greater risk for differential growth. It is no longer customary to wait until the child reaches school age or older because of functional and cosmetic factors.

76
Q

A neonate has severe swelling and congestion of the right hand; photographs are shown above. The right leg was autoamputated below the knee at birth. Which of the following is the most appropriate management?

(A) Observation
(B) Elevation of the hand and gentle compression
(C) Fasciotomies of the hand and extrinsic musculature
(D) Excision of the constricting band with interdigitating Z-plasty flaps
(E) Amputation

A

The correct response is Option D.

This neonate has amniotic constriction band syndrome that has resulted in severe edema of the hand, and urgent intervention is required because of the risk for vascular congestion and necrosis. Amniotic constriction bands are an embryologic anomaly with no genetic predisposition. The bands can be released partially or completely; partial release is advocated if additional vascular compromise is noted intraoperatively. In this patient, the constriction band has impinged all tissues to the skeletal structures, but complete excision is possible. Z-plasty is recommended for repair because of the severe circumferential size difference between the proximal and distal parts of the arm. This technique provides a greater surface area to allow establishment of lymphatic and venous drainage channels. In patients with less severe constriction banding, observation or delayed release is most appropriate.

The photograph above shows the results seven days after excision of the constricting band and multiple Z-plasties.
Observation without intervention is most likely to result in eventual autoamputation of the hand. Elevation of the hand will not relieve the severe congestion associated with this constriction band, and compression would further worsen the compromised vascular supply. Fasciotomies are contraindicated because this patient does not have signs consistent with compartment syndrome. Amputation is reserved for patients who have severe necrosis following failed attempts at hand salvage.

77
Q

A neonate has thumb hypoplasia and a shortened forearm with radial curvature. The child’s mother has a similar deformity. Which of the following is the most closely associated condition?

(A) Fanconi anemia
(B) Klippel-TrŽnaunay syndrome
(C) Maffucci syndrome
(D) Poland syndrome
(E) Sturge-Weber syndrome

A

The correct response is Option A.

This neonate has a radial club hand, an anomaly of unknown cause that may possibly be associated with autosomal dominant inheritance. Radial club hand deformities can range from partial underdevelopment to total absence of the radial elements of the upper extremity. Conditions associated with radial dysplasia include VATER syndrome (involving vertebral, anal, tracheoesophageal, and renal defects), Holt-Oram syndrome (involving cardiac septal defects), gastrointestinal disorders, and hematologic abnormalities including Fanconi anemia and thrombocytopenia-absent radius (TAR) syndrome.

Fanconi aplastic anemia is the most serious condition associated with radial club hand. Affected patients also have pancytopenia, a progressive condition that may not be detected initially. Mitomycin testing should be performed in neonates with radial club hand to detect Fanconi anemia and prevent the development of life-threatening complications.

Klippel-TrŽnaunay syndrome is characterized by musculoskeletal limb-length discrepancies involving one extremity, typically the leg, with associated venous and lymphatic malformations. Club hand is not seen.

Maffucci syndrome involves multiple enchondromas and venous and lymphatic anomalies. The limbs are usually foreshortened. Exostoses may be noted on the fingers.

Poland syndrome typically involves preaxial hypoplasia of the chest wall and upper extremity unilaterally. Affected patients have absence of the pectoralis major and minor muscles and limb hypoplasia with syndactyly and brachydactyly.

Patients with Sturge-Weber syndrome have port-wine stains in the distribution of the first and second divisions of the trigeminal nerve (V1and V2) associated with seizures and ipsilateral leptomeningeal calcifications.

78
Q

Which of the following is NOT a cause of the thumb-in-palm deformity in patients with cerebral palsy?

(A) Flaccidity of the extensor and abductor tendons of the thumb
(B) Hypomobility of the metacarpophalangeal joint of the thumb
(C) Skin contracture of the first web space
(D) Spasticity of an adductor tendon of the thumb
(E) Spasticity of a flexor tendon of the thumb

A

The correct response is Option A.

In patients with cerebral palsy, the thumb-in-palm deformity significantly limits function of the hand. The House classification is used to stratify the thumb-in-palm deformity according to four types. Type I is a simple adduction contracture of the metacarpal that results from spasticity of the adductor and first dorsal interosseous tendons. The type II deformity is similar to type I but also includes a flexion deformity of the metacarpophalangeal (MP) joint that is caused by spasticity of the flexor pollicis longus tendon. Patients with type III deformity have a simple adduction contracture of the metacarpal in conjunction with a hyperextension deformity and/or instability of the MP joint; this deformity results from spasticity of the adductor, first dorsal interosseous, and extensor pollicis brevis tendons and hyperextensibility of the MP joint. A type IV deformity is similar to type I deformity but also includes spasticity of the flexor pollicis longus tendon.

Treatment of the thumb-in-palm deformity involves release of the spastic muscles and stabilization of the joint.

Contracture of the first web space is also likely to exacerbate a thumb-in-palm deformity. Flaccidity of the extensor and abductor tendons of the thumb does not produce this deformity.

79
Q

A 3-year-old girl with constriction band syndrome has absence of functional fingers on the dominant right hand. The dominant thumb is completely spared. Which of the following is most appropriate for correction of this deformity?
(A) Intensive occupational therapy
(B) Use of digital prostheses
(C) Metacarpal lengthening
(D) Reconstruction with a radial forearm osteocutaneous flap
(E) Toe-to-hand autotransplantation

A

The correct response is Option E.

Reconstruction should be considered in a child who has partial or complete absence of function of the thumb or one or more fingers. Absence of functional fingers is an uncommon condition caused by either congenital amniotic band syndrome or a variant of hypoplasia or aplasia. This 3-year-old child who has congenital amniotic band syndrome is an excellent candidate for reconstruction because, in patients with this syndrome, the structures that exist proximal to the level of deformity (eg, bone, nerves, muscles, tendons, and vessels) are unaffected. Therefore, toe-to-hand transfer would be most appropriate because it would provide the best sensorimotor function and aesthetic outcome. Toes can be transferred independently or as a group and positioned for either fine motor or grasp functions. Complications in the donor foot are typically minor. The growth of the transferred toe has been estimated as 90% of a normal toe; there is a 10% risk for premature physeal closure.

Occupational therapy would yield only limited functional improvement in a patient who has no functional fingers. Digital prostheses would provide aesthetic but not functional improvement. Metacarpal lengthening is performed to provide a post for the thumb to work against; reconstruction with a radial forearm flap could also create a thumb post. Both techniques are inferior to an appropriately placed, rehabilitated toe transfer.

80
Q

A 1-year-old infant has the deformity shown in the photographs above. These findings are most consistent with which of the following?

(A) Arthrogryposis
(B) Complete thumb-index syndactyly
(C) Madelung’s deformity
(D) Poland’s syndrome
(E) Radial club hand

A

The correct response is Option E.

This infant has a radial club hand or radially deficient hand, involving partial or total absence of the radial or preaxial border of the upper extremity. Although radial club hand deformities can range from thumb hypoplasia to varying degrees of absence of the radius, resulting in radial deviation of the hand, most patients have total absence of the radius with absence of the radial carpal bones and thumb. It occurs in one in every 55,000 to 100,000 neonates and can be unilateral or bilateral. Abnormalities associated with radial dysplasia include cardiac septal defects (Holt-Oram syndrome), hematopoietic disorders (Fanconi anemia and TAR (thrombocytopenia-absent radius) syndrome), and gastrointestinal disorders (imperforate anus). Radial dysplasia may also occur as part of VATER syndrome (involving vertebral, anal, tracheoesophageal, and renal defects).

Fanconi aplastic anemia is the most serious condition associated with radial club hand. Affected patients also have pancytopenia, a progressive condition that may not be detected initially. Mitomycin testing should be performed in neonates with radial club hand to detect Fanconi anemia and prevent the development of life-threatening complications.

Arthrogryposis is a congenital disorder characterized by nonprogressive joint contractures present at birth. It is thought to occur secondary to neurogenic or myopathic weakness in utero. Multiple joints are typically affected; the extremities are atrophic and have limited active and passive motion. The wrist has a clublike appearance but is usually flexed and deviated ulnarly. Neurogenic and myopathic disorders are often associated with arthrogryposis.

Although this infant’s deformity may appear similar to thumb-index syndactyly, this is a rare finding and would not be characterized by radial deviation of the carpus and hypoplasia of the thenar eminence, which are seen in this patient.
Madelung’s deformity is a congenital disorder of the wrist that first becomes apparent in late childhood or adolescence. Affected patients have shortening of the radius, apparent palmar subluxation of the carpus, and prominence of the ulnar head. Finger abnormalities are not associated.

Patients with Poland’s syndrome have unilateral findings including absence of the pectoralis major and minor muscles, hypoplasia of the breast, and preaxial deformities.

81
Q

A 5-year-old child has an angular deformity of the ring finger; a photograph and radiograph are shown above. This finding is most consistent with which of the following?

(A) Complex incomplete syndactyly
(B) Fracture malunion
(C) Longitudinally bracketed epiphysis
(D) Madelung’s deformity
(E) Osteoma of the proximal phalanx

A

The correct response is Option C.

These findings are most consistent with a longitudinally bracketed epiphysis. In this child’s ring finger, there is a bracketed longitudinal growth plate with a reverse C-shaped configuration, rather than a normal transverse growth plate configuration. Longitudinal growth on the radial aspect of the phalanx is arrested but proceeds more normally on the ulnar aspect of the proximal phalanx, where the epiphysis is not bracketed. This finger deformity is most frequently associated with a delta phalanx, which is indeed present in this patient. However, a delta phalanx is more commonly evident in triphalangeal thumbs.

This child underwent release of simple, incomplete syndactyly of the third postaxial web space at age 8 months but subsequently developed a progressive angular deformity of the ring finger and severe scissoring beneath the long finger, as shown in the photograph. Angulation of greater than 5 degrees in any of the digits should be corrected surgically. Closing wedge osteotomy and epiphyseolysis with fat interposition were performed in this patient.

Complex incomplete syndactyly is characterized by webbing of the bone and soft tissues between adjacent digits that does not extend the length of the digits. Fracture malunion can be easily recognized on radiographs but is not seen in this patient. Patients with Madelung’s deformity have arrested growth of the ulnar aspect of the distal radius resulting in a patterned wrist deformity. Osteoma of the proximal phalanx appears as a smooth, oval radiolucency of bone on radiographs.

82
Q

A 1-year-old infant has the malformation shown in the photograph above. Which of the following is most likely in this infant?

(A) Apert’s syndrome
(B) Carpenter’s syndrome
(C) Pfeiffer’s syndrome
(D) Poland’s syndrome
(E) No other associated syndromes

A

The correct response is Option E.

This infant with syndactyly will most likely have no other associated syndromes. Syndactyly occurs in one of every 2000 neonates and is typically an isolated deformity, although other malformations have been associated. Syndactyly can be complete or incomplete, reflecting the level of webbing distally to proximally, and complex or simple, reflecting the presence or absence of bony union. Complicated syndactyly implies involvement of other congenital abnormalities such as polydactyly.

Male infants are more commonly affected than female infants. Inheritance is more often sporadic than familial. Approximately 50% of affected patients have bilateral syndactyly. The third web space is affected most frequently.

Patients with Apert’s syndrome have bilateral, symmetric, complex syndactyly with shortened fingers. Craniofacial deformities are characteristic. Carpenter’s syndrome is characterized by simple syndactyly, shortened fingers, preaxial polysyndactyly, broad thumbs, and craniosynostosis. In Pfeiffer’s syndrome, there is mild partial syndactyly, as well as broadened, deviated thumbs and severe midface hypoplasia. Poland’s syndrome is associated with unilateral findings, including absence of the sternocostal portion of the pectoralis major muscle, hypoplasia of the upper extremity, simple syndactyly, and shortened fingers.

83
Q

Which of the following hand deformities is commonly associated with a delta phalanx?

(A) Brachydactyly
(B) Camptodactyly
(C) Clinodactyly
(D) Macrodactyly
(E) Symbrachydactyly

A

The correct response is Option C.

Clinodactyly, or “bent finger,” is most commonly caused by the presence of a delta phalanx. Clinodactyly and triphalangeal thumb are most often associated with the presence of a delta phalanx, which has an angulated, short, tubular base, trapezoidal or triangular in shape. The growth plate is longitudinal, often a reverse C-shape or bracket, rather than transverse. The longitudinally bracketed epiphysis can interfere with normal longitudinal growth. Clinodactyly manifests as a radially or ulnarly curved digit. It is typically bilateral and is often transmitted as an autosomal dominant trait with variable expression. Although any digit can be involved, radial deviation of the small finger in the area of the distal interphalangeal joint is most common. In patients with delta phalanges, appropriate management includes bracket resection, fat grafting, and osteotomies, which can be either closing, opening, or reverse, as indicated by the findings in the affected patient.

The other hand deformities listed above can be associated with abnormally shaped phalanges but not with a delta phalanx.

Brachydactyly is a congenital deformity, typically autosomal dominant, that is characterized by abnormal shortening of the digits.

Patients with camptodactyly have a flexion deformity of the proximal interphalangeal joint, most frequently involving the small finger. Possible causes of camptodactyly include skin deficiencies, shortening of the volar plate, abnormalities of the palmar fascia and Landsmeer’s ligament, deficiencies of the central slip, and shortening of the superficialis tendon. Static and dynamic splinting can be helpful; surgical correction is often incomplete.

Macrodactyly describes enlargement of all of the structures of a digit. Staged bony and soft-tissue reduction procedures can be performed to allow for some control. Attempting to halt digital growth with ligation of digital arteries or elastic compression is typically ineffective. Amputation is indicated for any large, unsightly digit that significantly impairs total hand function.

Symbrachydactyly is a congenital hand deformity characterized by shortened digits with syndactyly. Poland’s syndrome is frequently associated. The digits are foreshortened but not angulated.

84
Q

A 3-year-old girl has congenital amniotic band syndrome affecting the nondominant left hand. On examination of the hand, the thumb is intact and functional, but there is adactyly of the other digits at the level of the metacarpophalangeal joint. The right hand is unaffected.

In order to improve function of this patient’s left hand, which of the following is the most appropriate management?

(A) Use of digital prostheses
(B) Toe phalangeal grafting
(C) Metacarpal lengthening
(D) Second toe-to-hand transfer
(E) Thumb amputation

A

The correct response is Option D.

Although there are many options for reconstruction of this patient’s fingers, the second-toe-to-hand transfer will provide the greatest restoration of function. Congenital band syndrome is one of few conditions (along with variations of hypoplasia and aplasia) that can affect the fingers but leave a functional thumb. In fact, children with congenital band syndrome often have better hand function following reconstruction because the structures proximal to the level of the deformity are unaffected (ie, bones, muscles, tendons, and vessels).

Because these recipient structures are normal, a second toe-to-hand transfer will provide the best sensory, motor, and overall hand function. The transferred second toe can be positioned to provide either pincer or grasp functions. The transfer is successful in most cases, and growth of the transferred toe is approximately 90% of the normal toe. Donor site morbidity is rare. The incidence of premature physeal closure is 10%.

Digital prostheses will improve the aesthetic quality of the child’s hand but will not restore function. Toe phalangeal grafting is more appropriate for treatment of aphalangia or hypophalangia. These grafts can be used to improve motion at the metacarpophalangeal joints but have a poorer functional outcome when compared with the second toe transfer. Although metacarpal lengthening can be performed to produce a “post” for the thumb to work against, the resultant digit is also less useful than a transferred second toe. A functioning thumb should never be amputated.

85
Q

A 22-year-old man has had a progressively worsening fixed flexion contracture of the proximal interphalangeal (PIP) joint of the left little finger since birth. There is no history of trauma or illness. The patient has stiffness of the PIP joint but no pain in the finger.

These findings are most consistent with

(A) camptodactyly
(B) clinodactyly
(C) scleroderma
(D) symbrachydactyly
(E) systemic lupus erythematosus

A

The correct response is Option A.

This 22-year-old man has findings consistent with camptodactyly (“bent finger”), a nontraumatic flexion deformity of the proximal interphalangeal (PIP) joint. This condition can be either sporadic or autosomal dominant and occurs in less than 1% of the population. The little finger is most frequently affected, and nearly two thirds of patients with camptodactyly have bilateral findings. The underlying etiology of this deformity involves volar PIP joint structures and is thought to result from abnormal insertion of either the lumbrical or superficialis tendons. Static or dynamic splinting has been shown to be beneficial in some patients. Surgical repair often yields only partial correction.

Clinodactyly involves either a radial or ulnar curvature of the fingers. This condition is believed to be autosomal dominant, is often bilateral, and is typically associated with some forms of mental retardation, including Down syndrome. Severe clinodactyly is often seen in association with a delta phalanx. Excision and wedge osteotomy are recommended for correction.

Symbrachydactyly is a sporadic, inherited central hand defect characterized by shortened, syndactylous digits. This condition is unilateral. The index, middle, and ring fingers are typically affected.

Systemic conditions such as scleroderma (primary systemic sclerosis) and systemic lupus erythematosus can involve the interphalangeal joints; however, dorsal wounds and flexion contractures are typically associated.

86
Q

A 2-year-old girl with VATER syndrome (involving vertebral, anal, tracheoesophageal, radial, and renal defects) has total absence of the left radius (type IV radial club hand). Which of the following is the most appropriate management for stabilization of the wrist?

(A) Ablation of the ulna
(B) Centralization of the ulna
(C) Lengthening of the ulna
(D) Shortening of the ulna
(E) Straightening of the ulna

A

The correct response is Option B.

This 2-year-old girl has complete absence of the left radius (type IV radial club hand) involving an absence of the thumb associated with type IV radial dysplasia. The most appropriate management is centralization of the ulna into the carpus followed by pollicization of the index finger. Total absence is the most common radial deficiency. Radial dysplasia is seen in as many as one in 55,000 births; boys and girls are affected equally. This deformity is frequently bilateral; preaxial hypoplasia or aplasia is common, and the radial artery is usually absent. The thumb may be hypoplastic and is in fact absent in nearly half of all affected persons. A common radial muscle mass is noted. The wrist is angulated radially from 30 degrees to 90 degrees, the carpus articulates with the radial border of the distal ulna, and the scaphoid and trapezium are usually absent. In addition to VATER syndrome, conditions associated with radial dysplasia include Holt-Oram syndrome and TAR (thrombocytopenia-absent radius) syndrome.

Although centralization of the carpus is a technically difficult procedure, it has been shown to be successful in many patients with radial dysplasia. This technique aligns the hand over the distal ulna, resulting in improved hand function. Following centralization, pollicization of the index finger will help with grasping of large objects.

Corrective closing wedge ulnar osteotomy is often necessary for patients who have ulnar bowing of greater than 35 degrees. Other techniques such as ulnar lengthening, shortening, or ablation will not correct the alignment of the hand and wrist in relation to the forearm.

87
Q

During embryologic development, inhibition of physiologic cell death in the interdigital area will most likely result in the development of which of the following congenital hand deformities?

(A) Cleft hand
(B) Constriction band syndrome
(C) Phocomelia
(D) Polydactyly
(E) Syndactyly

A

The correct response is Option E.

During embryologic development of the upper limb, the lateral body wall elevates at approximately four weeks after conception and develops into a mass of undifferentiated mesenchyme covered by ectoderm known as the limb bud. The limb bud elongates beneath the apical ectodermal ridge, and its distal end expands into the hand plate at five weeks of gestation. During the next two weeks, physiologic cell death occurs between the digital rays, leading to separation of the digits and the formation of a normal human hand. In contrast, syndactyly, or fusion of the digits, results from inhibition of this cell death in the interdigital area.

Other abnormalities in development that occur during this time period can manifest as specific limb malformations. Cleft hand results from hypoplasia of the ectodermal ridge, while polydactyly is caused by hyperplasia of the ectodermal ridge. Constriction band syndrome occurs when amniotic bands constrict a digit or extremity, resulting in tissue loss and scarring.

Phocomelia is a congenital anomaly that involves gross defects of the bones of the upper extremity. Severe intercalary and/or longitudinal deficiencies are associated. In patients with this deformity, the hand may attach directly to the humerus or even to the trunk. The forearm may also attach directly to the trunk. Administration of thalidomide during pregnancy has been linked to the development of phocomelia in the fetus.

88
Q
A
89
Q

A 12-year-old boy is brought to the emergency department because he has had episodes of syncope for the past week. Physical examination of the hands shows hypoplasia of the left thumb. Electrocardiography shows atrial fibrillation. Which of the following is the most likely diagnosis?
A) Holt-Oram syndrome
B) Fanconi anemia
C) Robert syndrome
D) TAR syndrome
E) VATER association

A

The correct answer is a.
Holt-Oram syndrome is an autosomal-dominant disorder involving cardiac defects and upper limb anomalies. Common cardiac defects include atrial or ventricular septal defects and conduction abnormalities or blocks. Upper limb anomalies are variable in type and severity, ranging from subtle pre-axial carpal abnormalities to absent thumbs or radial club hand. The disorder is due to a mutation in a gene encoding a transcription factor involved in both cardiac and upper limb development.
Fanconi anemia is primarily characterized by bone marrow failure. It is an autosomal-recessive disease due to mutations in a series of genes believed to be involved in DNA repair. Many patients are born with a variety of radial limb anomalies, including hypoplastic thumbs or absent radii. Other common anomalies include café au lait spots, short stature, genital abnormalities, and microcephaly. Patients usually die of complications from pancytopenia, with a life expectancy of 20-30 years.
TAR syndrome involves thrombocytopenia and absent radii. It can be distinguished from Fanconi anemia by the neonatal onset of thrombocytopenia, which is typically seen later in life with Fanconi anemia, and the absence of pancytopenia.
Robert syndrome is a very rare disorder characterized by tetraphocomelia, similar to that seen with thalidomide embryopathy, as well as brain and craniofacial abnormalities.
VATER is an acronym for vertebral/vascular anomalies, anal atresia, tracheo-esophageal fistula, esophageal atresia, and renal/radial anomalies. Cardiac defects, other limb anomalies, and a single umbilical artery are also associated with this complex, giving rise to the alternative acronym VACTERLS.

90
Q

A 1-year-old boy is brought to the office by his parents for consultation of a flexion deformity of the right thumb. Physical examination shows fixed flexion of the thumb at the interphalangeal joint. No other abnormalities are noted. Which of the following is the most likely diagnosis?
A) Camptodactyly
B) Clinodactyly
C) Delta phalanx
D) Macrodactyly
E) Trigger thumb

A

The correct answer is option E.

Trigger digit reportedly occurs with an incidence of 1 in 2000 to 3 in 1000 births. Whereas most authorities believe this deformity is a congenital abnormality, there is debate whether trigger digit is an acquired deformity. The congenital trigger finger presents as a flexed digit either at birth or up to 3 years of age. Often it may not be recognized early because of the tendencies of infants to hold their fingers in a flexed position. Before 12 months of age, observation is the standard treatment. After 3 years of age, surgical release is generally recommended. The results of conservative nonoperative treatment and spontaneous recovery have been reported as between 0% and 30%.
Trigger digit can be confused with the diagnosis of camptodactyly, which is a congenital flexion posture most often found in the small finger. Although congenital trigger thumb is related to a nodule in the tendon, the etiology of camptodactyly is less clear, with various etiologies related to skin deficit, volar plate abnormalities, and abnormalities of the lumbrical and superficialis tendons.
Abnormally angulated digits in the radial ulnar plane is clinodactyly. Clinodactyly may be associated with a delta phalanx, which is an abnormally shaped, short tubular bone within the digit.
Macrodactyly refers to an abnormal enlargement of the digit. In true macrodactyly, all structures of the digits are enlarged.

91
Q

A 2-year-old boy is referred to the office by a pediatrician for surgical correction of a deformity of the right hand. Physical examination shows Wassel Type 4 duplication of the thumb. Which of these is a standard component of the surgical management of the condition?
A) Amputation of the ulnar duplication
B) Maintenance of the ulnar collateral ligament at the metacarpophalangeal joint
C) Repair of the flexor pollicis longus tendon
D) Repair of the nail bed

A

The correct answer is option B.
There are seven classes of thumb duplication as described by Wassel. Type 4 is the most common. Type 1 involves a duplication of the distal phalanx in which the two are connected at the base. Type 2 duplication occur when the two distal phalanges are separate. Type 3 duplication are of the proximal phalanx and are connected at the base. Type 4 duplication are of the distal and proximal phalanxes when there are four distal and proximal phalanges. Type 5 are duplication of the distal and proximal phalanxes with a duplication of the metacarpal still fused at the base. Type 6 duplication result in six separate bones, two distal phalanges, two proximal phalanges, and two metacarpals that are separate.

In the correction of Type 4 duplication, it is most critical to preserve the ulnar collateral ligament to stabilize pinch. Typically, the radial thumb is less functional and less developed. The radial collateral ligament (RCL) is reconstructed with a ligament-periosteal flap. The abductor pollicis brevis is included in this RCL reconstruction. Most often, the flexor pollicis longus in the ulnar element is intact and does not require repair or reconstruction. As seen in the photograph of the thumb duplication in the patient described, the nail complex does not require intervention.

92
Q

A 3-month-old infant is brought to the office for consultation regarding deformity of the right hand (shown). The infant is otherwise healthy and has no other abnormalities. On physical examination, no abnormalities other than those of the right hand are noted. Radiographs show no bony syndactyly. The most appropriate management of this patient’s deformity is release of the syndactyly and which of the following?

A) First dorsal interosseus flap
B) Full-thickness skin grafting
C) Radial artery flap
D) Split-thickness skin grafting

A

The correct answer is option B.
Syndactyly is the second most common congenital hand abnormality after polydactyly, occurring in 1 in 2000 live births. Syndactyly can be described as complete, when the interdigital web extends to the full length of the digit, or incomplete. Although there have been reports of the use of skin flaps or primary closure after radical subcutaneous tissue debulking, skin grafts are almost always necessary. Full-thickness skin grafting is the appropriate management of syndactyly in a child. Effective grafts for this procedure include the lateral groin crease, plantar foot, antecubital fossa, inner arm, or wrist flexion crease.
Amputation of the digits is almost never required for a simple syndactyly.
Pedicle flaps such as a dorsal interosseus flap or radial artery flap are not required because tissue can be closed with adjacent flaps and skin grafts alone.
A split-thickness skin graft is a poor choice in a young child because it is difficult to obtain and has a significant amount of contracture.

93
Q

A 6 year old boy is brought to the office because of enlargement of the index and long fingers that has been progressing since infancy. A photograph of the fingers is shown. He has not had pain. Two point discrimination is 6 to 8 mm in the affected digits. Which of the following is the most appropriate treatment?

A) Administration of thyroid supplements
B) Radiation therapy to the fingers
C) Ray amputation of the index finger
D) Surgical debulking and epiphysiodesis
E) Systemic administration of oral corticosteroids

A

The correct answer is option D.

The photograph shows a case of macrodactyly of both the index and long fingers. This deformity is caused by the development of a lipomatous hamartoma usually within the digital nerves with resulting overgrowth of the hand and fingers within the distribution of the tumor. If the affected fingers are sensate, then debulking procedures, in addition to epiphysiodesis (premature closure of the growth plates), at the distal interphalangeal, proximal interphalangeal, and metacarpophalangeal joints may slow progression of the disease process. Epiphysiodesis should be performed when the fingers are at the adult length.
Radiation and thyroid supplementation are not known to affect the growth of such tumors and carry significant side effects. The affected digits are often insensate and stiff with cold intolerance. When indicated by painful, functionally massive overgrowth, amputation is an option, although generally not initially. Systemic administration of corticosteroids is the treatment for some forms of congenital hemangiomas.

94
Q

A 7-year-old girl is brought to the office by her parents because she has a deformity of the left hand. Medical history is unknown because the patient was recently adopted. She is otherwise healthy. Examination of the right hand shows no abnormalities. Her parents say that she is able to play with her toys with both hands and can pick up small objects with the left hand. A photograph of the hands is shown. Which of the following is the most appropriate option for surgical reconstruction?

A) Distraction lengthening of the thumb
B) Microvascular transfer of second and third toes to the index and long finger positions
C) Microvascular transfer of second and third toes to the small and ring finger positions
D) Transfer of toe phalangeal epiphysis to the index and long finger positions
E) No surgical reconstruction

A

Correct answer is option E.

Congenital hand deformities present a challenge to the patient, parents, and the reconstructive surgeon. Transverse arrest or congenital amputations may present as a failure of formation or arrest of development of the limb. Operative indications for this group of patients are often limited but are driven by the function of the child. The patient described is a very functional child who uses her limb for complex grasping activities; therefore, no surgery is indicated.
Distraction lengthening of the thumb is not appropriate for the patient described since it may impede pulp-to-pulp opposition of the radial and ulnar digits. Microvascular transfer of one or two toes may be a worthwhile procedure in children with no opposable digits and no functional use of their hand. However, in the patient described, this procedure is not appropriate because function is already satisfactory. Toe epiphysis nonvascular transfers have been indicated in congenitally short digits, but for the patient described there are no normal phalanges to warrant this procedure.

95
Q

Hematologic evaluation is indicated before surgical intervention for which of the following conditions?
A) Atypical cleft hand
B) Constriction band syndrome
C) Radial hypoplasia
D) Small finger polydactyly
E) Thumb duplication

A

Correct answer is option C.

Radial aplasia and hypoplasia are associated with several syndromes including VACTERL (vertebral, anal, cardiac, tracheoesophageal, renal/radial, limb), Holt Oram, TAR (thrombocytopenia absent radius), and Fanconi anemia. Both TAR syndrome and Fanconi anemia are contraindications for early surgical intervention until the underlying hematologic abnormalities have stabilized or been corrected.
The other congenital disorders listed tend to occur sporadically or familially and are not associated with specific hematologic abnormalities.

96
Q

A male newborn has the isolated congenital deformities of the left hand shown in the photographs. Which of these surgical options may improve the function of the hand?

A) Amputation of the long, ring, and small fingers
B) Full-thickness matrix grafting
C) Phalangeal lengthening of the small and ring fingers
D) Radial metacarpal lengthening of the index and long fingers
E) Toe-to-hand transplantation

A

The correct answer is option C.

The patient described has an isolated congenital anomaly of the left hand known as shortened fingers or brachydactyly. The most functional improvement is to lengthen the small and ring fingers to facilitate power grip and opposition. One method is nonvascularized toe phalanx grafting; another is distraction lengthening. This procedure should be performed when the patient is between 8 and 12 months of age.
Amputation would not add to the patient’s function. Full-thickness matrix grafting is for reconstruction of the nail matrix complex. Lengthening the second and third metacarpals would not address the functional deficit. The patient’s thumb appears to be quite functional, so toe-to-hand transplantation is not appropriate.

97
Q

In a child with Type III B hypoplastic thumb, the most appropriate thumb reconstruction is…
A) ablation and index finger pollicization.
B) amputation.
C) basilar joint reconstruction.
D) distraction osteogenesis lengthening.
E) Huber transfer.

A

The correct option is a.
 The five categories in the classification of congenital hypoplastic thumb range from Type I, for a small thumb with all essential components, to Type V, for complete absence of the thumb. Type III is subdivided into type A, for intact carpometacarpal joint, and type B, for absence of the joint.
Because of the instability associated with absence of the carpometacarpal joint, the most appropriate reconstruction of Type III B deformity is ablation of the thumb and pollicization of the index finger.
The goal of hypoplastic thumb reconstruction is a functional thumb. Amputation, distraction lengthening, and basilar joint reconstruction do not result in functional thumb. Furthermore, adequate autologous elements are not present for basilar joint reconstruction.
A Huber transfer is used to create functional opposition when absent. Type III A deformities can be reconstructed with the Huber transfer, which involves transfer of the hypothenar muscle to recreate the thenar musculature.

98
Q

A 2-month-old infant is brought to the office for evaluation of bilateral complex syndactyly of the hands and feet. Which of the following is the most likely gene involved?

a. Engrailed-1 (EN1)
b. Fibroblast growth factor receptor-2 (FGFR2)
c. LIM Homeobox transcription factor-1 beta (LMX1B)
d. Sonic hedgehog (SHH)
e. Wingless type (Wnt)

A

The correct response is Option B.

This patient presents with complex syndactyly of hands and feet consistent with Apert syndrome. This is an autosomal dominant condition that can be caused by mutation in the fibroblast growth factor receptor-2 gene.

Limb development is controlled by signaling pathways that are located within three different signaling centers. These govern the proximal-distal, anterior-posterior, and dorsal-ventral axes of limb development.

The limb develops in a proximal-distal direction, from shoulder to wrist, which is controlled by the apical ectodermal ridge (AER), a thickened layer of ectoderm over the limb bud. Fibroblast growth factors secreted within the AER signal the underlying mesoderm to differentiate. Disruption of the AER results in truncation of the limb.

Growth in the anterior-posterior (radioulnar) axis is determined by the zone of polarizing activity (ZPA). This is located in the posterior margin of the limb bud. The sonic hedgehog protein (SHH) acts to signal development of the limb into radial and ulnar aspects. Alterations in this pathway can result in mirror duplication.

Dorsal-ventral limb development is signaled by the Wnt signaling pathway, which produces a transcription factor, LMX1B, that induces the development of dorsal structures. Deficiency in LMX1B has been associated with nail-patella syndrome. In the ventral portion of the limb, the Engrailed-1 gene product blocks the Wnt pathway, leading to ventralization.

A variety of other transcription factors encoded by Hox and T-Box genes also govern limb and organ development, and alterations in these can give rise to various developmental differences.

Reference(s)

  1. Kay SP, McCombe DB, Kozin S. Deformities of the Hand and Fingers. In: Wolfe SW, Hotchkiss RN, Pedersen WC, et al. Green’s Operative Hand Surgery, 7th ed. Philadelphia, PA: Elsevier Churchill Livingstone; 2016;36:1217-1288.
  2. Kozin S. Embryology of the Upper Extremity. In: Wolfe SW, Hotchkiss RN, Pedersen WC, et al. Green’s Operative Hand Surgery, 7th ed. Philadelphia, PA: Elsevier Churchill Livingstone; 2016;35:1208-1216.
  3. Oberg KC, Feenstra JM, Manske P, et al. Developmental Biology and Classification of Congenital Anomalies of the Hand and Upper Extremity. J Hand Surg. 2010;35A:2066-2076.
  4. Oberg KC, Tonkin MA. Embryology and Classification. In: Weiss APC, Goldfarb CA, Hentz VR, et al. Textbook of Hand and Upper Extremity Surgery. Chicago, IL: American Society for Surgery of the Hand; 2013;53.
  5. Oda T, Pushman, AG, Chung KC. Treatment of Common Congenital Hand Conditions. Plast Reconstr Surg. 2010;126:121e-133e.